TMYN Logo Drawing

The Math You Need, When You Need It

math tutorials for students majoring in the earth sciences

Scientific Notation - Practice Problems

Solving earth science problems with scientific notation, × div[id^='image-'] {position:static}div[id^='image-'] div.hover{position:static} introductory problems.

These problems cover the fundamentals of writing scientific notation and using it to understand relative size of values and scientific prefixes.

Problem 1: The distance to the moon is 238,900 miles. Write this value in scientific notation.

Problem 2: One mile is 1609.34 meters. What is the distance to the moon in meters using scientific notation?

`1609.34 m/(mi) xx 238","900 mi` = 384,400,000 m

Notice in the above unit conversion the 'mi' units cancel each other out because 'mi' is in the denominator for the first term and the numerator for the second term

Earth from space

Problem 4: The atomic radius of a magnesium atom is approximately 1.6 angstroms, which is equal to 1.6 x 10 -10 meters (m). How do you write this length in standard form?

 0.00000000016 m  

Fissure A = 40,0000 m Fissure B = 5,0000 m

This shows fissure A is larger (by almost 10 times!). The shortcut to answer a question like this is to look at the exponent. If both coefficients are between 1-10, then the value with the larger exponent is the larger number.

Problem 6: The amount of carbon in the atmosphere is 750 petagrams (pg). One petagram equals 1 x 10 15 grams (g). Write out the amount of carbon in the atmosphere in (i) scientific notation and (ii) standard decimal format.

The exponent is a positive number, so the decimal will move to the right in the next step.

750,000,000,000,000,000 g

Advanced Problems

Scientific notation is used in solving these earth and space science problems and they are provided to you as an example. Be forewarned that these problems move beyond this module and require some facility with unit conversions, rearranging equations, and algebraic rules for multiplying and dividing exponents. If you can solve these, you've mastered scientific notation!

Problem 7: Calculate the volume of water (in cubic meters and in liters) falling on a 10,000 km 2 watershed from 5 cm of rainfall.

`10,000  km^2 = 1 xx 10^4  km^2`

5 cm of rainfall = `5 xx 10^0 cm`

Let's start with meters as the common unit and convert to liters later. There are 1 x 10 3 m in a km and area is km x km (km 2 ), therefore you need to convert from km to m twice:

`1 xx 10^3 m/(km) * 1 xx 10^3 m/(km) = 1 xx 10^6 m^2/(km)^2` `1 xx 10 m^2/(km)^2 * 1 xx 10^4 km^2 = 1 xx 10^10 m^2` for the area of the watershed.

For the amount of rainfall, you should convert from centimeters to meters:

`5 cm * (1 m)/(100 cm)= 5 xx 10^-2 m`

`V = A * d`

When multiplying terms with exponents, you can multiply the coefficients and add the exponents:

`V = 1 xx 10^10 m^2 * 5.08 xx 10^(-2) m = 5.08 xx 10^8 m^3`

Given that there are 1 x 10 3 liters in a cubic meter we can make the following conversion:

`1 xx 10^3 L * 5.08 xx 10^8 m^3 = 5.08 xx 10^11 L`

Step 5. Check your units and your answer - do they make sense?           

`V = 4/3 pi r^3`

Using this equation, plug in the radius (r) of the dust grains.

`V = 4/3 pi (2 xx10^(-6))^3m^3`

Notice the (-6) exponent is cubed. When you take an exponent to an exponent, you need to multiply the two terms

`V = 4/3 pi (8 xx10^(-18)m^3)`

Then, multiple the cubed radius times pi and 4/3

`V = 3.35 xx 10^(-17) m^3`

`m = 3.35 xx 10^(-17) m^3 * 3300 (kg)/m^3`

Notice in the equation above that the m 3 terms cancel each other out and you are left with kg

`m = 1.1 xx 10^(-13) kg`

Barnard nebula

`V = 4/3 pi (2.325 xx10^(15) m)^3`

`V = 5.26 xx10^(46) m^3`

Number of dust grains = `5.26 xx10^(46) m^3 xx 0.001` grains/m 3

Number of dust grains = `5.26xx10^43 "grains"`

Total mass = `1.1xx10^(-13) (kg)/("grains") * 5.26xx10^43 "grains"`

Notice in the equation above the 'grains' terms cancel each other out and you are left with kg

Total mass = `5.79xx10^30 kg`

If you feel comfortable with this topic, you can go on to the assessment . Or you can go back to the Scientific Notation explanation page .

« Previous Page       Next Page »

Library homepage

  • school Campus Bookshelves
  • menu_book Bookshelves
  • perm_media Learning Objects
  • login Login
  • how_to_reg Request Instructor Account
  • hub Instructor Commons
  • Download Page (PDF)
  • Download Full Book (PDF)
  • Periodic Table
  • Physics Constants
  • Scientific Calculator
  • Reference & Cite
  • Tools expand_more
  • Readability

selected template will load here

This action is not available.

Mathematics LibreTexts

4.4: Scientific Notation

  • Last updated
  • Save as PDF
  • Page ID 51461

Learning Objectives

  • Define decimal and scientific notation

Convert between scientific and decimal notation

  • Multiply and divide numbers expressed in scientific notation
  • Solve application problems involving scientific notation

Before we can convert between scientific and decimal notation, we need to know the difference between the two. S cientific notation is used by scientists, mathematicians, and engineers when they are working with very large or very small numbers. Using exponential notation, large and small numbers can be written in a way that is easier to read.

When a number is written in scientific notation, the exponent tells you if the term is a large or a small number. A positive exponent indicates a large number and a negative exponent indicates a small number that is between 0 and 1. It is difficult to understand just how big a billion or a trillion is. Here is a way to help you think about it.

1 billion can be written as 1,000,000,000 or represented as \(2\times10^9\).

A light year is the number of miles light travels in one year, about 5,880,000,000,000. That’s a lot of zeros, and it is easy to lose count when trying to figure out the place value of the number. Using scientific notation, the distance is \(5\times10^{-8}\) mm. In this case the \(-8\) tells us how many places to count to the right of the decimal.

Outlined in the box below are some important conventions of scientific notation format.

Scientific Notation

A positive number is written in scientific notation if it is written as \(1\leq{a}<10\), and n is an integer.

Look at the numbers below. Which of the numbers is written in scientific notation?

Now let’s compare some numbers expressed in both scientific notation and standard decimal notation in order to understand how to convert from one form to the other. Take a look at the tables below. Pay close attention to the exponent in the scientific notation and the position of the decimal point in the decimal notation.

Convert from decimal notation to scientific notation

To write a large number in scientific notation, move the decimal point to the left to obtain a number between 1 and 10. Since moving the decimal point changes the value, you have to multiply the decimal by a power of 10 so that the expression has the same value.

Let’s look at an example.

\(\begin{array}{r}180,000.=18,000.0\times10^{1}\\1,800.00\times10^{2}\\180.000\times10^{3}\\18.0000\times10^{4}\\1.80000\times10^{5}\\180,000=1.8\times10^{5}\end{array}\)

Notice that the decimal point was moved 5 places to the left, and the exponent is 5.

Write the following numbers in scientific notation.

  • \(920,000,000\)
  • \(10,200,000\)
  • \(100,000,000,000\)

[reveal-answer q=”628″]Show Solution[/reveal-answer] [hidden-answer a=”628″]

  • \(\underset{\longleftarrow}{920,000,000}=920,000,000.0\), move the decimal point 8 times to the left and you will have \(9.2\times10^{8}\)
  • \(\underset{\longleftarrow}{10,200,000}=10,200,000.0=1.02\times10^{7}\), note here how we included the 0 and the 2 after the decimal point. In some disciplines, you may learn about when to include both of these. Follow instructions from your teacher on rounding rules.
  • \(\underset{\longleftarrow}{100,000,000,000}=100,000,000,000.0=1.0\times10^{11}\)

[/hidden-answer]

To write a small number (between 0 and 1) in scientific notation, you move the decimal to the right and the exponent will have to be negative, as in the following example.

\(\begin{array}{r}\underset{\longrightarrow}{0.00004}=00.0004\times10^{-1}\\000.004\times10^{-2}\\0000.04\times10^{-3}\\00000.4\times10^{-4}\\000004.\times10^{-5}\\0.00004=4\times10^{-5}\end{array}\)

You may notice that the decimal point was moved five places to the right until you got to the number 4, which is between 1 and 10. The exponent is \(−5\).

  • \(0.0000000000035\)
  • \(0.0000000102\)
  • \(0.00000000000000793\)

[reveal-answer q=”229054″]Show Solution[/reveal-answer] [hidden-answer a=”229054″]

  • \(\underset{\longrightarrow}{0.0000000000035}=3.5\times10^{-12}\), we moved the decimal 12 times to get to a number between 1 and 10
  • \(\underset{\longrightarrow}{0.0000000102}=1.02\times10^{-8}\)
  • \(\underset{\longrightarrow}{0.00000000000000793}=7.93\times10^{-15}\)

In the following video you are provided with examples of how to convert both a large and a small number in decimal notation to scientific notation.

Thumbnail for the embedded element "Examples: Write a Number in Scientific Notation"

A YouTube element has been excluded from this version of the text. You can view it online here: pb.libretexts.org/ba/?p=94

Convert from scientific notation to decimal notation

You can also write scientific notation as decimal notation. Recall the number of miles that light travels in a year is \(5\times10^{-8}\) mm. To write each of these numbers in decimal notation, you move the decimal point the same number of places as the exponent. If the exponent is positive , move the decimal point to the right. If the exponent is negative , move the decimal point to the left.

\(\begin{array}{l}5.88\times10^{12}=\underset{\longrightarrow}{5.880000000000.}=5,880,000,000,000\\5\times10^{-8}=\underset{\longleftarrow}{0.00000005.}=0.00000005\end{array}\)

For each power of 10, you move the decimal point one place. Be careful here and don’t get carried away with the zeros—the number of zeros after the decimal point will always be 1 less than the exponent because it takes one power of 10 to shift that first number to the left of the decimal.

Write the following in decimal notation.

  • \(4.8\times10{-4}\)
  • \(3.08\times10^{6}\)

[reveal-answer q=”489774″]Show Solution[/reveal-answer] [hidden-answer a=”489774″]

  • \(\underset{\longleftarrow}{4.8\times10^{-4}}=\underset{\longleftarrow}{.00048}\)
  • \(\underset{\longrightarrow}{3.08\times10^{6}}=\underset{\longrightarrow}{3080000}\)

Think About It

To help you get a sense of the relationship between the sign of the exponent and the relative size of a number written in scientific notation, answer the following questions. You can use the textbox to wirte your ideas before you reveal the solution.

1. You are writing a number that is greater than 1 in scientific notation. Will your exponent be positive or negative?

[practice-area rows=”1″][/practice-area]

2. You are writing a number that is between 0 and 1 in scientific notation. Will your exponent be positive or negative?

3. What power do you need to put on 10 to get a result of 1?

[practice-area rows=”1″][/practice-area] [reveal-answer q=”824936″]Show Solution[/reveal-answer] [hidden-answer a=”824936″] 1. You are writing a number that is greater than 1 in scientific notation. Will your exponent be positive or negative? For numbers greater than 1, the exponent on 10 will be positive when you are using scientific notation. Refer to the table presented above:

2. You are writing a number that is between 0 and 1 in scientific notation. Will your exponent be positive or negative? We can reason that since numbers greater than 1 will have a positive exponent, numbers between 0 and 1 will have a negative exponent. Why are we specifying numbers between 0 and 1? The numbers between 0 and 1 represent amounts that are fractional. Recall that we defined numbers with a negative exponent as \(10^{-2}\) we have \(\frac{1}{10\times10}=\frac{1}{100}\) which is a number between 0 and 1.

3. What power do you need to put on 10 to get a result of 1? Recall that any number or variable with an exponent of 0 is equal to 1, as in this example:

\(\begin{array}{c}\frac{t^{8}}{t^{8}}=\frac{\cancel{t^{8}}}{\cancel{t^{8}}}=1\\\frac ParseError: EOF expected (click for details) Callstack: at (Courses/Lumen_Learning/Beginning_Algebra_(Lumen)/04:_Exponents/4.04:_Scientific_Notation), /content/body/div[2]/div[3]/div[2]/div/p[10]/span[1], line 1, column 2 ParseError: EOF expected (click for details) Callstack: at (Courses/Lumen_Learning/Beginning_Algebra_(Lumen)/04:_Exponents/4.04:_Scientific_Notation), /content/body/div[2]/div[3]/div[2]/div/p[10]/span[2], line 1, column 2 ={t}^{8 - 8}={t}^{0}\\\text{ therefore }\\{t}^{0}=1\end{array}\)

We now have described the notation necessary to write all possible numbers on the number line in scientific notation.

In the next video you will see how to convert a number written in scientific notation into decimal notation.

Thumbnail for the embedded element "Examples: Writing a Number in Decimal Notation When Given in Scientific Notation"

Multiplying and Dividing Numbers Expressed in Scientific Notation

Numbers that are written in scientific notation can be multiplied and divided rather simply by taking advantage of the properties of numbers and the rules of exponents that you may recall. To multiply numbers in scientific notation, first multiply the numbers that aren’t powers of 10 (the a in \(a\times10^{n}\)). Then multiply the powers of ten by adding the exponents.

This will produce a new number times a different power of 10. All you have to do is check to make sure this new value is in scientific notation. If it isn’t, you convert it.

Let’s look at some examples.

\(\left(3\times10^{8}\right)\left(6.8\times10^{-13}\right)\)

[reveal-answer q=”395606″]Show Solution[/reveal-answer] [hidden-answer a=”395606″]Regroup using the commutative and associative properties.

\(\left(3\times6.8\right)\left(10^{8}\times10^{-13}\right)\)

Multiply the coefficients.

\(\left(20.4\right)\left(10^{8}\times10^{-13}\right)\)

Multiply the powers of 10 using the Product Rule. Add the exponents.

\(20.4\times10^{-5}\)

Convert 20.4 into scientific notation by moving the decimal point one place to the left and multiplying by \(10^{1}\).

\(\left(2.04\times10^{1}\right)\times10^{-5}\)

Group the powers of 10 using the associative property of multiplication.

\(2.04\times\left(10^{1}\times10^{-5}\right)\)

Multiply using the Product Rule—add the exponents.

\(2.04\times10^{1+\left(-5\right)}\)

\(\left(3\times10^{8}\right)\left(6.8\times10^{-13}\right)=2.04\times10^{-4}\)

\(\left(8.2\times10^{6}\right)\left(1.5\times10^{-3}\right)\left(1.9\times10^{-7}\right)\)

[reveal-answer q=”23947″]Show Solution[/reveal-answer] [hidden-answer a=”23947″]Regroup using the commutative and associative properties.

\(\left(8.2\times1.5\times1.9\right)\left(10^{6}\times10^{-3}\times10^{-7}\right)\)

Multiply the numbers.

\(\left(23.37\right)\left(10^{6}\times10^{-3}\times10^{-7}\right)\)

Multiply the powers of 10 using the Product Rule—add the exponents.

\(23.37\times10^{-4}\)

Convert 23.37 into scientific notation by moving the decimal point one place to the left and multiplying by \(10^{1}\).

\(\left(2.337\times10^{1}\right)\times10^{-4}\)

\(2.337\times\left(10^{1}\times10^{-4}\right)\)

Multiply using the Product Rule and add the exponents.

\(2.337\times10^{1+\left(-4\right)}\)

\(\left(8.2\times10^{6}\right)\left(1.5\times10^{-3}\right)\left(1.9\times10^{-7}\right)=2.337\times10^{-3}\)

In the following video you will see an example of how to multiply tow numbers that are written in scientific notation.

Thumbnail for the embedded element "Examples: Multiplying Numbers Written in Scientific Notation"

In order to divide numbers in scientific notation, you once again apply the properties of numbers and the rules of exponents. You begin by dividing the numbers that aren’t powers of 10 (the a in \(a\times10^{n}\). Then you divide the powers of ten by subtracting the exponents.

This will produce a new number times a different power of 10. If it isn’t already in scientific notation, you convert it, and then you’re done.

\(\displaystyle \frac{2.829\times 1 ParseError: EOF expected (click for details) Callstack: at (Courses/Lumen_Learning/Beginning_Algebra_(Lumen)/04:_Exponents/4.04:_Scientific_Notation), /content/body/div[3]/div[3]/p[1]/span[1], line 1, column 2 }{3.45\times 1 ParseError: EOF expected (click for details) Callstack: at (Courses/Lumen_Learning/Beginning_Algebra_(Lumen)/04:_Exponents/4.04:_Scientific_Notation), /content/body/div[3]/div[3]/p[1]/span[2], line 1, column 2 }\)

[reveal-answer q=”364796″]Show Solution[/reveal-answer] [hidden-answer a=”364796″]Regroup using the associative property.

\(\displaystyle \left( \frac{2.829}{3.45} \right)\left( \frac ParseError: colon expected (click for details) Callstack: at (Courses/Lumen_Learning/Beginning_Algebra_(Lumen)/04:_Exponents/4.04:_Scientific_Notation), /content/body/div[3]/div[3]/p[3]/span[1], line 1, column 4 ParseError: colon expected (click for details) Callstack: at (Courses/Lumen_Learning/Beginning_Algebra_(Lumen)/04:_Exponents/4.04:_Scientific_Notation), /content/body/div[3]/div[3]/p[3]/span[2], line 1, column 4 \right)\)

Divide the coefficients.

\(\displaystyle \left(0.82\right)\left( \frac ParseError: colon expected (click for details) Callstack: at (Courses/Lumen_Learning/Beginning_Algebra_(Lumen)/04:_Exponents/4.04:_Scientific_Notation), /content/body/div[3]/div[3]/p[5]/span[1], line 1, column 4 ParseError: colon expected (click for details) Callstack: at (Courses/Lumen_Learning/Beginning_Algebra_(Lumen)/04:_Exponents/4.04:_Scientific_Notation), /content/body/div[3]/div[3]/p[5]/span[2], line 1, column 4 \right)\)

Divide the powers of 10 using the Quotient Rule. Subtract the exponents.

\(\begin{array}{l}0.82\times10^{-9-\left(-3\right)}\\0.82\times10^{-6}\end{array}\)

Convert 0.82 into scientific notation by moving the decimal point one place to the right and multiplying by \(10^{-1}\).

\(\left(8.2\times10^{-1}\right)\times10^{-6}\)

Group the powers of 10 together using the associative property.

\(8.2\times\left(10^{-1}\times10^{-6}\right)\)

\(8.2\times10^{-1+\left(-6\right)}\)

\(\displaystyle \frac{2.829\times ParseError: EOF expected (click for details) Callstack: at (Courses/Lumen_Learning/Beginning_Algebra_(Lumen)/04:_Exponents/4.04:_Scientific_Notation), /content/body/div[3]/div[3]/div/p[1]/span[1], line 1, column 3 }{3.45\times ParseError: EOF expected (click for details) Callstack: at (Courses/Lumen_Learning/Beginning_Algebra_(Lumen)/04:_Exponents/4.04:_Scientific_Notation), /content/body/div[3]/div[3]/div/p[1]/span[2], line 1, column 3 }=8.2\times ParseError: EOF expected (click for details) Callstack: at (Courses/Lumen_Learning/Beginning_Algebra_(Lumen)/04:_Exponents/4.04:_Scientific_Notation), /content/body/div[3]/div[3]/div/p[1]/span[3], line 1, column 3 \)

\(\displaystyle \frac{\left(1.37\times10^{4}\right)\left(9.85\times10^{6}\right)}{5.0\times10^{12}}\)

[reveal-answer q=”337143″]Show Solution[/reveal-answer] [hidden-answer a=”337143″]Regroup the terms in the numerator according to the associative and commutative properties.

\(\displaystyle \frac{\left( 1.37\times 9.85 \right)\left( ParseError: EOF expected (click for details) Callstack: at (Courses/Lumen_Learning/Beginning_Algebra_(Lumen)/04:_Exponents/4.04:_Scientific_Notation), /content/body/div[3]/div[4]/p[3]/span[1], line 1, column 3 \times ParseError: EOF expected (click for details) Callstack: at (Courses/Lumen_Learning/Beginning_Algebra_(Lumen)/04:_Exponents/4.04:_Scientific_Notation), /content/body/div[3]/div[4]/p[3]/span[2], line 1, column 3 \right)}{5.0\times ParseError: EOF expected (click for details) Callstack: at (Courses/Lumen_Learning/Beginning_Algebra_(Lumen)/04:_Exponents/4.04:_Scientific_Notation), /content/body/div[3]/div[4]/p[3]/span[3], line 1, column 3 }\)

\(\displaystyle \frac{13.4945\times ParseError: EOF expected (click for details) Callstack: at (Courses/Lumen_Learning/Beginning_Algebra_(Lumen)/04:_Exponents/4.04:_Scientific_Notation), /content/body/div[3]/div[4]/p[5]/span[1], line 1, column 3 }{5.0\times ParseError: EOF expected (click for details) Callstack: at (Courses/Lumen_Learning/Beginning_Algebra_(Lumen)/04:_Exponents/4.04:_Scientific_Notation), /content/body/div[3]/div[4]/p[5]/span[2], line 1, column 3 }\)

Regroup using the associative property.

\(\displaystyle \left( \frac{13.4945}{5.0} \right)\left( \frac ParseError: colon expected (click for details) Callstack: at (Courses/Lumen_Learning/Beginning_Algebra_(Lumen)/04:_Exponents/4.04:_Scientific_Notation), /content/body/div[3]/div[4]/p[7]/span[1], line 1, column 4 ParseError: colon expected (click for details) Callstack: at (Courses/Lumen_Learning/Beginning_Algebra_(Lumen)/04:_Exponents/4.04:_Scientific_Notation), /content/body/div[3]/div[4]/p[7]/span[2], line 1, column 4 \right)\)

Divide the numbers.

\(\displaystyle \left(2.6989\right)\left(\frac{10^{10}}{10^{12}}\right)\)

Divide the powers of 10 using the Quotient Rule—subtract the exponents.

\(\displaystyle \begin{array}{c}\left(2.6989 \right)\left( ParseError: EOF expected (click for details) Callstack: at (Courses/Lumen_Learning/Beginning_Algebra_(Lumen)/04:_Exponents/4.04:_Scientific_Notation), /content/body/div[3]/div[4]/p[11]/span[1], line 1, column 3 \right)\\2.6989\times ParseError: EOF expected (click for details) Callstack: at (Courses/Lumen_Learning/Beginning_Algebra_(Lumen)/04:_Exponents/4.04:_Scientific_Notation), /content/body/div[3]/div[4]/p[11]/span[2], line 1, column 3 \end{array}\)

\(\displaystyle \frac{\left( 1.37\times ParseError: EOF expected (click for details) Callstack: at (Courses/Lumen_Learning/Beginning_Algebra_(Lumen)/04:_Exponents/4.04:_Scientific_Notation), /content/body/div[3]/div[4]/div/p[1]/span[1], line 1, column 3 \right)\left( 9.85\times ParseError: EOF expected (click for details) Callstack: at (Courses/Lumen_Learning/Beginning_Algebra_(Lumen)/04:_Exponents/4.04:_Scientific_Notation), /content/body/div[3]/div[4]/div/p[1]/span[2], line 1, column 3 \right)}{5.0\times ParseError: EOF expected (click for details) Callstack: at (Courses/Lumen_Learning/Beginning_Algebra_(Lumen)/04:_Exponents/4.04:_Scientific_Notation), /content/body/div[3]/div[4]/div/p[1]/span[3], line 1, column 3 }=2.6989\times ParseError: EOF expected (click for details) Callstack: at (Courses/Lumen_Learning/Beginning_Algebra_(Lumen)/04:_Exponents/4.04:_Scientific_Notation), /content/body/div[3]/div[4]/div/p[1]/span[4], line 1, column 3 \)

Notice that when you divide exponential terms, you subtract the exponent in the denominator from the exponent in the numerator. You will see another example of dividing numbers written in scientific notation in the following video.

Thumbnail for the embedded element "Examples: Dividing Numbers Written in Scientific Notation"

Solve application problems

Molecule of water with one oxygen bonded to two hydrogen.

Learning rules for exponents seems pointless without context, so let’s explore some examples of using scientific notation that involve real problems. First, let’s look at an example of how scientific notation can be used to describe real measurements.

Match each length in the table with the appropriate number of meters described in scientific notation below.

[reveal-answer q=”993302″]Show Solution[/reveal-answer] [hidden-answer a=”993302″]

Red Blood Cells.

One of the most important parts of solving a “real” problem is translating the words into appropriate mathematical terms, and recognizing when a well known formula may help. Here’s an example that requires you to find the density of a cell, given its mass and volume. Cells aren’t visible to the naked eye, so their measurements, as described with scientific notation, involve negative exponents.

Human cells come in a wide variety of shapes and sizes. The mass of an average human cell is about \(10^{-6}\text{ meters }^3\). [3] Biologists have recently discovered how to use the density of some types of cells to indicate the presence of disorders such as sickle cell anemia or leukemia. [4] Density is calculated as the ratio of \(\frac{\text{ mass }}{\text{ volume }}\). Calculate the density of an average human cell.

[reveal-answer q=”856454″]Show Solution[/reveal-answer] [hidden-answer a=”856454″]

Read and Understand: We are given an average cellular mass and volume as well as the formula for density. We are looking for the density of an average human cell.

Define and Translate: \(v=\text{volume}=10^{-6}\text{ meters}^3\), \(\text{density}=\frac{\text{ mass }}{\text{ volume }}\)

Write and Solve: Use the quotient rule to simplify the ratio.

\(\begin{array}{c}\text{ density }=\frac{2\times10^{-11}\text{ grams }}{10^{-6}\text{ meters }^3}\\\text{ }\\\,\,\,\,\,\,\,\,\,\,\,\,\,\,\,\,\,\,\,\,\,\,\,\,\,\,\,\,\,\,\,\,\,\,\,\,\,=2\times10^{-11-\left(-6\right)}\frac{\text{ grams }}{\text{ meters }^3}\\\text{ }\\\,\,\,\,\,\,\,\,\,\,\,\,\,\,\,\,\,\,\,\,\,\,\,=2\times10^{-5}\frac{\text{ grams }}{\text{ meters }^3}\\\end{array}\)

If scientists know the density of healthy cells, they can compare the density of a sick person’s cells to that to rule out or test for disorders or diseases that may affect cellular density.

The average density of a human cell is \(2\times10^{-5}\frac{\text{ grams }}{\text{ meters }^3}\)

The following video provides an example of how to find the number of operations a computer can perform in a very short amount of time.

Thumbnail for the embedded element "Application of Scientific Notation - Quotient 2 (Time for Computer Operations)"

In the next example, you will use another well known formula, \(d=r\cdot{t}\), to find how long it takes light to travel from the sun to the earth. Unlike the previous example, the distance between the earth and the sun is massive, so the numbers you will work with have positive exponents.

The speed of light is \(1.5\times10^{11}\) meters from earth, how many seconds does it take for sunlight to reach the earth? Write your answer in scientific notation. [reveal-answer q=”532092″]Show Solution[/reveal-answer] [hidden-answer a=”532092″]

Read and Understand: We are looking for how long—an amount of time. We are given a rate which has units of meters per second and a distance in meters. This is a \(d=r\cdot{t}\) problem.

Define and Translate:

\(\begin{array}{l}d=1.5\times10^{11}\\r=3\times10^{8}\frac{\text{ meters }}{\text{ second }}\\t=\text{ ? }\end{array}\)

Write and Solve: Substitute the values we are given into the \(d=r\cdot{t}\) equation. We will work without units to make it easier. Often, scientists will work with units to make sure they have made correct calculations.

\(\begin{array}{c}d=r\cdot{t}\\1.5\times10^{11}=3\times10^{8}\cdot{t}\end{array}\)

Divide both sides of the equation by \(3\times10^{8}\) to isolate t.

\(\begin{array}{c}1.5\times10^{11}=3\times10^{8}\cdot{t}\\\text{ }\\\frac{1.5\times10^{11}}{3\times10^{8}}=\frac{3\times10^{8}}{3\times10^{8}}\cdot{t}\end{array}\)

On the left side, you will need to use the quotient rule of exponents to simplify, and on the right, you are left with t.

\(\begin{array}{c}\frac{1.5\times10^{11}}{3\times10^{8}}=\frac{3\times10^{8}}{3\times10^{8}}\cdot{t}\\\text{ }\\\left(\frac{1.5}{3}\right)\times\left(\frac{10^{11}}{10^{8}}\right)=t\\\text{ }\\\left(0.5\right)\times\left(10^{11-8}\right)=t\\0.5\times10^3=t\end{array}\)

This answer is not in scientific notation, so we will move the decimal to the right, which means we need to subtract one factor of 10.

\(0.5\times10^3=5.0\times10^2=t\)

The time it takes light to travel from the sun to the earth is \(5.0\times10^2=t\) seconds, or in standard notation, 500 seconds. That’s not bad considering how far it has to travel!

In the following video we calculate how many miles the participants of the New York marathon ran combined, and compare that to the circumference of the earth.

Thumbnail for the embedded element "Application of Scientific Notation - Quotient 1 (Number of Times Around the Earth)"

Large and small numbers can be written in scientific notation to make them easier to understand. In the next section, you will see that performing mathematical operations such as multiplication and division on large and small numbers is made easier by scientific notation and the rules of exponents.

Scientific notation was developed to assist mathematicians, scientists, and others when expressing and working with very large and very small numbers. Scientific notation follows a very specific format in which a number is expressed as the product of a number greater than or equal to one and less than ten, and a power of 10. The format is written \(1\leq{a}<10\) and n is an integer. To multiply or divide numbers in scientific notation, you can use the commutative and associative properties to group the exponential terms together and apply the rules of exponents.

  • Orders of magnitude (mass). (n.d.). Retrieved May 26, 2016, from https://en.Wikipedia.org/wiki/Orders_of_magnitude_(mass) ↵
  • How Big is a Human Cell? ↵
  • How big is a human cell? - Weizmann Institute of Science. (n.d.). Retrieved May 26, 2016, from www.weizmann.ac.il/plants/Milo/images/humanCellSize120116Clean.pdf↵
  • Grover, W. H., Bryan, A. K., Diez-Silva, M., Suresh, S., Higgins, J. M., & Manalis, S. R. (2011). Measuring single-cell density. Proceedings of the National Academy of Sciences, 108(27), 10992-10996. doi:10.1073/pnas.1104651108 ↵
  • Revision and Adaptation. Provided by : Lumen Learning. License : CC BY: Attribution
  • Application of Scientific Notation - Quotient 1 (Number of Times Around the Earth). Authored by : James Sousa (Mathispower4u.com) for Lumen Learning. Located at : https://youtu.be/15tw4-v100Y . License : CC BY: Attribution
  • Application of Scientific Notation - Quotient 2 (Time for Computer Operations). Authored by : James Sousa (Mathispower4u.com) for Lumen Learning. Located at : https://youtu.be/Cbm6ejEbu-o . License : CC BY: Attribution
  • Screenshot: water molecule. Provided by : Lumen Learning. License : CC BY: Attribution
  • Screenshot: red blood cells. Provided by : Lumen Learning. License : CC BY: Attribution
  • Screenshot: light traveling from the sun to the earth. Provided by : Lumen Learning. License : CC BY: Attribution
  • Unit 11: Exponents and Polynomials, from Developmental Math: An Open Program. Provided by : Monterey Institute of Technology and Education. Located at : nrocnetwork.org/resources/downloads/nroc-math-open-textbook-units-1-12-pdf-and-word-formats/. License : CC BY: Attribution
  • Examples: Write a Number in Scientific Notation. Authored by : James Sousa (Mathispower4u.com) for Lumen Learning. Located at : https://youtu.be/fsNu3AdIgdk . License : CC BY: Attribution
  • Examples: Writing a Number in Decimal Notation When Given in Scientific Notation. Authored by : James Sousa (Mathispower4u.com) for Lumen Learning. Located at : https://youtu.be/8BX0oKUMIjw . License : CC BY: Attribution
  • Examples: Dividing Numbers Written in Scientific Notation. Authored by : James Sousa (Mathispower4u.com) for Lumen Learning. Located at : https://youtu.be/RlZck2W5pO4 . License : CC BY: Attribution
  • Examples: Multiplying Numbers Written in Scientific Notation. Authored by : James Sousa (Mathispower4u.com) . Located at : https://youtu.be/5ZAY4OCkp7U . License : CC BY: Attribution

Scientific Notation Questions with Solutions

Questions on scientific notation are presented along with answers and detailed solutions . Scientific notation of a number is of form: \( a \times 10^n \) where \( 1 \le a \lt 10 \), \( n \) is an integer and \( a \) is called the mantissa.

  •   \( 5.9 \times 10^{-6} - 2.0 \times 10^{-7} = \) A) \( \quad 5.7 \times 10^{-7} \) B) \( \quad 5.7 \times 10^{-6} \) C) \( \quad 3.9 \times 10^{-6} \) D) \( \quad 7.9 \times 10^{-6} \) E) \( \quad 5.7 \times 10^{-5} \)

Solutions to the Above Questions

  •  Answer E Solution In \( a \), the mantissa \( 23.7 \) is greater than \(10 \), therefore it is not in scientific notation. In \( c \), the base \( 5 \) must be base \( 10 \). In \( d \), the mantissa \( 0.3 \) is less than \(1 \), therefore it is not in scientific notation. In \( e \), the mantissa \( 10.0 \) is equal to \(10 \), therefore it is not in scientific notation.
  •  Answer E Solution Scientific notation of a number is of form: \( a \times 10^n \) where \( 1 \le a \lt 10 \) and \( n \) is an integer. \( 510000 \) is bigger than \( 10 \), therefore start with a decimal point from the right \( 510000. \) and move it till the number is between \( 1 \) and \( 10 \) not included. Hence for 510000, if we start with a decimal point from the right, we need to move the decimal point n = 5 times in order to obtain \( 5.10000 \) which is a number between \( 1 \) and \( 10 \) not included. \( 510,000 \) is written in scientific notation as: \( 5.1 \times 10^{5} \)
  •  Answer A Solution We first add the numbers: \( 100000 + 3000000 = 3100000 \) \( 3100000 \) is bigger than 10, we therefore start with a decimal point on the right and move it \( n = 6 \) times to obtain \( 3.1 \) which is a number between \( 1 \) and \( 10 \) not included. Hence \( 100000 + 3,000000 = 3.1 \times 10^{6} \)
  •  Answer C Solution The given number \( 0.0000028 \) is smaller than one and in order to obtain a number between \( 1 \) and \( 10 \) not included, we need to move the decimal point to the RIGHT. We need to move the decimal point \( n = 6 \) times in order to write the given number as \( 2.8 \), between \( 1 \) and \( 10 \) not included Hence \( 0.0000028 \) is written in scientific notation as: \( 2.8 \times 10^{-6} \)
  •  Answer D Solution In this question, we are given the scientific notation of a number and asked to write it in standard form. Hence \( 1.2 \times 10^6 = 1.2 \times 1000000 = 1200000 \)
  •  Answer B Solution We are given the scientific notation: \( 2.3 \times 10^{-5} = \dfrac{2.3}{100000} = 0.000023 \)
  •  Answer E Solution Write in decimal form: \( \quad \dfrac{1}{10000}+\dfrac{2}{1000000} = 0.0001 + 0.000002 = 0.000102 \) In scientific notation: \( \quad \dfrac{1}{10000}+\dfrac{2}{1000000} = 0.000102 = 1.02 \times 10^{-4} \)
  •  Answer A Solution Write in decimal form: \( \quad 2.1 \times 10^{-5} + 3.0 \times 10^{-4} = 0.000021 + 0.0003 = 0.000321 \) In scientific notation: \( \quad 2.1 \times 10^{-5} + 3.0 \times 10^{-4} = 0.000021 + 0.0003 = 0.000321 = 3.21 \times 10^{-4} \)
  •  Answer C Solution The mantissan \( 120.054 \) is larger than \( 1 \), hence we need to move the decimal point \( n = 2 \) to the left to rewrite \( 120.054 \) in decimal form as \( 1.20054 \times 10^2 \) Hence \( \quad 120.054 \times 10^{-6} = 1.20054 \times 10^2 \times 10^{-6} = 1.20054 \times 10^{-4} \)
  •  Answer B Solution Write in decimal form: \( \quad 5.9 \times 10^{-6} - 2.0 \times 10^{-7} = 0.0000059 - 0.0000002 = 0.0000057 \) In scientific notation: \( \quad 0.0000057 = 5.7 \times 10^{-6} \)

HIGH SCHOOL

  • ACT Tutoring
  • SAT Tutoring
  • PSAT Tutoring
  • ASPIRE Tutoring
  • SHSAT Tutoring
  • STAAR Tutoring

GRADUATE SCHOOL

  • MCAT Tutoring
  • GRE Tutoring
  • LSAT Tutoring
  • GMAT Tutoring
  • AIMS Tutoring
  • HSPT Tutoring
  • ISAT Tutoring
  • SSAT Tutoring

Search 50+ Tests

Loading Page

math tutoring

  • Elementary Math
  • Pre-Calculus
  • Trigonometry

science tutoring

Foreign languages.

  • Mandarin Chinese

elementary tutoring

  • Computer Science

Search 350+ Subjects

  • Video Overview
  • Tutor Selection Process
  • Online Tutoring
  • Mobile Tutoring
  • Instant Tutoring
  • How We Operate
  • Our Guarantee
  • Impact of Tutoring
  • Reviews & Testimonials
  • Media Coverage
  • About Varsity Tutors

HiSET: Math : Solve problems using scientific notation

Study concepts, example questions & explanations for hiset: math, all hiset: math resources, example questions, example question #1 : solve problems using scientific notation.

Simplify the following expression using scientific notation.

scientific notation problem solving questions

You can solve this problem in several ways. One way is to convert each number out of scientific notation and write it out fully, then find the sum of the two values and convert the answer back into scientific notation.

scientific notation problem solving questions

Another, potentially faster, way to solve this problem is to convert one answer into the same scientific-notational terms as the other and then sum them.

scientific notation problem solving questions

Example Question #2 : Solve Problems Using Scientific Notation

Multiply, and express the product in scientific notation:

scientific notation problem solving questions

Convert 7,200,000 to scientific notation as follows:

Move the (implied) decimal point until it is immediately after the first nonzero digit (the 7). This required moving the point six units to the left:

scientific notation problem solving questions

Rearrange and regroup the expressions so that the powers of ten are together:

scientific notation problem solving questions

Multiply the numbers in front. Also, multiply the powers of ten by adding exponents:

scientific notation problem solving questions

In order for the number to be in scientific notation, the number in front must be between 1 and 10. An adjustment must be made by moving the implied decimal point in 36 one unit left. It follows that

scientific notation problem solving questions

the correct response.

scientific notation problem solving questions

Express the product in scientific notation.

scientific notation problem solving questions

None of the other choices gives the correct response.

scientific notation problem solving questions

Scientific notation refers to a number expressed in the form

scientific notation problem solving questions

Each factor can be rewritten in scientific notation as follows:

scientific notation problem solving questions

Now, substitute:

scientific notation problem solving questions

Apply the Product of Powers Property:

scientific notation problem solving questions

This is in scientific notation and is the correct choice.

Display vt optimized

Report an issue with this question

If you've found an issue with this question, please let us know. With the help of the community we can continue to improve our educational resources.

DMCA Complaint

If you believe that content available by means of the Website (as defined in our Terms of Service) infringes one or more of your copyrights, please notify us by providing a written notice (“Infringement Notice”) containing the information described below to the designated agent listed below. If Varsity Tutors takes action in response to an Infringement Notice, it will make a good faith attempt to contact the party that made such content available by means of the most recent email address, if any, provided by such party to Varsity Tutors.

Your Infringement Notice may be forwarded to the party that made the content available or to third parties such as ChillingEffects.org.

Please be advised that you will be liable for damages (including costs and attorneys’ fees) if you materially misrepresent that a product or activity is infringing your copyrights. Thus, if you are not sure content located on or linked-to by the Website infringes your copyright, you should consider first contacting an attorney.

Please follow these steps to file a notice:

You must include the following:

A physical or electronic signature of the copyright owner or a person authorized to act on their behalf; An identification of the copyright claimed to have been infringed; A description of the nature and exact location of the content that you claim to infringe your copyright, in \ sufficient detail to permit Varsity Tutors to find and positively identify that content; for example we require a link to the specific question (not just the name of the question) that contains the content and a description of which specific portion of the question – an image, a link, the text, etc – your complaint refers to; Your name, address, telephone number and email address; and A statement by you: (a) that you believe in good faith that the use of the content that you claim to infringe your copyright is not authorized by law, or by the copyright owner or such owner’s agent; (b) that all of the information contained in your Infringement Notice is accurate, and (c) under penalty of perjury, that you are either the copyright owner or a person authorized to act on their behalf.

Send your complaint to our designated agent at:

Charles Cohn Varsity Tutors LLC 101 S. Hanley Rd, Suite 300 St. Louis, MO 63105

Or fill out the form below:

Contact Information

Complaint details.

Learning Tools by Varsity Tutors

  • + ACCUPLACER Mathematics
  • + ACT Mathematics
  • + AFOQT Mathematics
  • + ALEKS Tests
  • + ASVAB Mathematics
  • + ATI TEAS Math Tests
  • + Common Core Math
  • + DAT Math Tests
  • + FSA Tests
  • + FTCE Math
  • + GED Mathematics
  • + Georgia Milestones Assessment
  • + GRE Quantitative Reasoning
  • + HiSET Math Exam
  • + HSPT Math
  • + ISEE Mathematics
  • + PARCC Tests
  • + Praxis Math
  • + PSAT Math Tests
  • + PSSA Tests
  • + SAT Math Tests
  • + SBAC Tests
  • + SIFT Math
  • + SSAT Math Tests
  • + STAAR Tests
  • + TABE Tests
  • + TASC Math
  • + TSI Mathematics
  • + ACT Math Worksheets
  • + Accuplacer Math Worksheets
  • + AFOQT Math Worksheets
  • + ALEKS Math Worksheets
  • + ASVAB Math Worksheets
  • + ATI TEAS 6 Math Worksheets
  • + FTCE General Math Worksheets
  • + GED Math Worksheets
  • + 3rd Grade Mathematics Worksheets
  • + 4th Grade Mathematics Worksheets
  • + 5th Grade Mathematics Worksheets
  • + 6th Grade Math Worksheets
  • + 7th Grade Mathematics Worksheets
  • + 8th Grade Mathematics Worksheets
  • + 9th Grade Math Worksheets
  • + HiSET Math Worksheets
  • + HSPT Math Worksheets
  • + ISEE Middle-Level Math Worksheets
  • + PERT Math Worksheets
  • + Praxis Math Worksheets
  • + PSAT Math Worksheets
  • + SAT Math Worksheets
  • + SIFT Math Worksheets
  • + SSAT Middle Level Math Worksheets
  • + 7th Grade STAAR Math Worksheets
  • + 8th Grade STAAR Math Worksheets
  • + THEA Math Worksheets
  • + TABE Math Worksheets
  • + TASC Math Worksheets
  • + TSI Math Worksheets
  • + AFOQT Math Course
  • + ALEKS Math Course
  • + ASVAB Math Course
  • + ATI TEAS 6 Math Course
  • + CHSPE Math Course
  • + FTCE General Knowledge Course
  • + GED Math Course
  • + HiSET Math Course
  • + HSPT Math Course
  • + ISEE Upper Level Math Course
  • + SHSAT Math Course
  • + SSAT Upper-Level Math Course
  • + PERT Math Course
  • + Praxis Core Math Course
  • + SIFT Math Course
  • + 8th Grade STAAR Math Course
  • + TABE Math Course
  • + TASC Math Course
  • + TSI Math Course
  • + Number Properties Puzzles
  • + Algebra Puzzles
  • + Geometry Puzzles
  • + Intelligent Math Puzzles
  • + Ratio, Proportion & Percentages Puzzles
  • + Other Math Puzzles

How to Solve Scientific Notation? (+FREE Worksheet!)

Learn how to solve mathematics problems containing scientific notation in a few simple and easy steps.

How to Solve Scientific Notation? (+FREE Worksheet!)

Related Topics

  • How to Solve Powers of Products and Quotients
  • How to Multiply Exponents
  • How to Divide Exponents
  • How to Solve Zero and Negative Exponents
  • How to Solve Negative Exponents and Negative Bases

Step by step guide to solve scientific notation problems

  • Scientific notation is used to write very big or very small numbers in decimal form.
  • In scientific notation all numbers are written in the form of: \(m×10^n\)

Scientific Notation – Example 1:

Write \(0.00015\) in scientific notation.

First, move the decimal point to the right so that you have a number that is between \(1\) and \(10\). Then: \(N=1.5\) Second, determine how many places the decimal moved in step \(1\) by the power of \(10\). Then: \(10^{ \ -4} →\) When the decimal is moved to the right, the exponent is negative. Then: \(0.00015=1.5×10^{ \ -4} \)

The Absolute Best Books to Ace Pre-Algebra to Algebra II

The Ultimate Algebra Bundle From Pre-Algebra to Algebra II

Scientific notation – example 2:.

Write \(9.5 \times 10^{\ -5}\) in standard notation.

\(10^{-5} →\) When the decimal is moved to the right, the exponent is negative. Then: \(9.5×10^{-5}=0.000095\)

Scientific Notation – Example 3:

Write \(0.00012\) in scientific notation.

First, move the decimal point to the right so that you have a number that is between \(1\) and \(10\). Then: \(N=1.2\) Second, determine how many places the decimal moved in step \(1\) by the power of \(10\). Then: \(10^{-4}→ \) When the decimal is moved to the right, the exponent is negative. Then: \(0.00012=1.2×10^{-4}\)

Scientific Notation – Example 4:

Write \(8.3×10^{-5}\) in standard notation.

\(10^{-5} →\) When the decimal is moved to the right, the exponent is negative. Then: \(8.3×10^{-5}=0.000083\)

Exercises for Solving Scientific Notation

Write each number in scientific notation..

  • \(\color{blue}{91 × 10^3}\)
  • \(\color{blue}{60}\)
  • \(\color{blue}{2000000}\)
  • \(\color{blue}{0.0000006}\)
  • \(\color{blue}{354000}\)
  • \(\color{blue}{0.000325}\)

Download Scientific Notation Worksheet

  • \(\color{blue}{9.1 × 10^4}\)
  • \(\color{blue}{6 × 10^1}\)
  • \(\color{blue}{2 × 10^6}\)
  • \(\color{blue}{6 × 10^{–7}}\)
  • \(\color{blue}{3.54 × 10^5}\)
  • \(\color{blue}{3.25 × 10^{–4}}\)

The Greatest Books for Students to Ace the Algebra

Pre-Algebra Exercise Book A Comprehensive Workbook + PreAlgebra Practice Tests

Pre-algebra in 10 days the most effective pre-algebra crash course, college algebra practice workbook the most comprehensive review of college algebra, high school algebra i a comprehensive review and step-by-step guide to mastering high school algebra 1, 10 full length clep college algebra practice tests the practice you need to ace the clep college algebra test.

by: Effortless Math Team about 4 years ago (category: Articles , Free Math Worksheets )

Effortless Math Team

Related to this article, more math articles.

  • FREE 3rd Grade MCAS Math Practice Test
  • The Law of Cosines
  • How to Make Inferences from Data? (+FREE Worksheet!)
  • Number Properties Puzzle – Challenge 14
  • Top 10 Tips to Create a PSAT/NMSQT Math Study Plan
  • The Ultimate 6th Grade RICAS Math Course (+FREE Worksheets)
  • The Ultimate Accuplacer Math Course (+FREE Worksheets)
  • Understanding Quadrants
  • The Ultimate GED Math Course [Updated for 2024]
  • How to Graph an Equation in Point-Slope Form?

What people say about "How to Solve Scientific Notation? (+FREE Worksheet!) - Effortless Math: We Help Students Learn to LOVE Mathematics"?

No one replied yet.

Leave a Reply Cancel reply

You must be logged in to post a comment.

Pre-Algebra Practice Workbook The Most Comprehensive Review of Pre-Algebra

Algebra i practice workbook the most comprehensive review of algebra 1, algebra ii practice workbook the most comprehensive review of algebra 2, algebra i for beginners the ultimate step by step guide to acing algebra i, algebra ii for beginners the ultimate step by step guide to acing algebra ii, pre-algebra for beginners the ultimate step by step guide to preparing for the pre-algebra test, pre-algebra tutor everything you need to help achieve an excellent score.

  • ATI TEAS 6 Math
  • ISEE Upper Level Math
  • SSAT Upper-Level Math
  • Praxis Core Math
  • 8th Grade STAAR Math

Limited time only!

Save Over 45 %

It was $89.99 now it is $49.99

Login and use all of our services.

Effortless Math services are waiting for you. login faster!

Study Guides > ALGEBRA / TRIG I

Problem solving with scientific notation, learning outcomes.

  • Solve application problems involving scientific notation

Molecule of water with one oxygen bonded to two hydrogen.

Solve application problems

Think about it.

Red Blood Cells.

[latex]\begin{array}{c}\text{ density }=\frac{2\times10^{-11}\text{ grams }}{10^{-6}\text{ meters }^3}\\\text{ }\\\,\,\,\,\,\,\,\,\,\,\,\,\,\,\,\,\,\,\,\,\,\,\,\,\,\,\,\,\,\,\,\,\,\,\,\,\,=2\times10^{-11-\left(-6\right)}\frac{\text{ grams }}{\text{ meters }^3}\\\text{ }\\\,\,\,\,\,\,\,\,\,\,\,\,\,\,\,\,\,\,\,\,\,\,\,=2\times10^{-5}\frac{\text{ grams }}{\text{ meters }^3}\\\end{array}[/latex]

The average density of a human cell is [latex]2\times10^{-5}\frac{\text{ grams }}{\text{ meters }^3}[/latex]

Earth in the foreground, sun in the background, light beams traveling from the sun to the earth.

[latex]\begin{array}{l}d=1.5\times10^{11}\\r=3\times10^{8}\frac{\text{ meters }}{\text{ second }}\\t=\text{ ? }\end{array}[/latex]

Write and Solve:  Substitute the values we are given into the [latex]d=r\cdot{t}[/latex] equation. We will work without units to make it easier. Often, scientists will work with units to make sure they have made correct calculations.

[latex]\begin{array}{c}d=r\cdot{t}\\1.5\times10^{11}=3\times10^{8}\cdot{t}\end{array}[/latex]

Divide both sides of the equation by [latex]3\times10^{8}[/latex] to isolate  t.

[latex]\begin{array}{c}1.5\times10^{11}=3\times10^{8}\cdot{t}\\\text{ }\\\frac{1.5\times10^{11}}{3\times10^{8}}=\frac{3\times10^{8}}{3\times10^{8}}\cdot{t}\end{array}[/latex]

On the left side, you will need to use the quotient rule of exponents to simplify, and on the right, you are left with  t. 

[latex]\begin{array}{c}\frac{1.5\times10^{11}}{3\times10^{8}}=\frac{3\times10^{8}}{3\times10^{8}}\cdot{t}\\\text{ }\\\left(\frac{1.5}{3}\right)\times\left(\frac{10^{11}}{10^{8}}\right)=t\\\text{ }\\\left(0.5\right)\times\left(10^{11-8}\right)=t\\0.5\times10^3=t\end{array}[/latex]

This answer is not in scientific notation, so we will move the decimal to the right, which means we need to subtract one factor of [latex]10[/latex].

[latex]0.5\times10^3=5.0\times10^2=t[/latex]

Contribute!

Licenses & attributions, cc licensed content, original.

  • Revision and Adaptation. Provided by: Lumen Learning License: CC BY: Attribution .
  • Application of Scientific Notation - Quotient 1 (Number of Times Around the Earth). Authored by: James Sousa (Mathispower4u.com) for Lumen Learning. License: CC BY: Attribution .
  • Application of Scientific Notation - Quotient 2 (Time for Computer Operations). Authored by: James Sousa (Mathispower4u.com) for Lumen Learning. License: CC BY: Attribution .
  • Screenshot: water molecule. Provided by: Lumen Learning License: CC BY: Attribution .
  • Screenshot: red blood cells. Provided by: Lumen Learning License: CC BY: Attribution .
  • Screenshot: light traveling from the sun to the earth. Provided by: Lumen Learning License: CC BY: Attribution .

CC licensed content, Shared previously

  • Unit 11: Exponents and Polynomials, from Developmental Math: An Open Program. Provided by: Monterey Institute of Technology and Education Located at: https://www.nroc.org/. License: CC BY: Attribution .

Please add a message.

Message received. Thanks for the feedback.

scientific notation problem solving questions

  • Parallelogram
  • Quadrilateral
  • Parallelepiped
  • Tetrahedron
  • Dodecahedron
  • Fraction Calculator
  • Mixed Fraction Calculator
  • Greatest Common Factor Calulator
  • Decimal to Fraction Calculator
  • Whole Numbers
  • Rational Numbers
  • Place Value
  • Irrational Numbers
  • Natural Numbers
  • Binary Operation
  • Numerator and Denominator
  • Order of Operations (PEMDAS)

Scientific Notation

Table of Contents

Last modified on August 3rd, 2023

#ezw_tco-2 .ez-toc-title{ font-size: 120%; ; ; } #ezw_tco-2 .ez-toc-widget-container ul.ez-toc-list li.active{ background-color: #ededed; } chapter outline

Scientific notation is a special way of representing numbers which are too large or small in a unique way that makes it easier to remember and compare them. They are expressed in the form (a × 10n). Here ‘a’ is the coefficient, and ‘n’ is the power or exponent of the base 10.

The diagram below shows the standard form of writing numbers in scientific notation:

scientific notation problem solving questions

Thus, scientific notation is a floating-point system where numbers are expressed as products consisting of numbers between 1 and 10 multiplied with appropriate power of 10. It helps to represent big and small numbers in a much easier way.

The speed of light(c) measured in a vacuum is approximately 300,000,000 meters per second, which is written as 3 × 10 8 m/s in scientific notation. Again the mass of the sun is written as 1.988 × 10 30 kg. All these values, if written in scientific form, will reduce a lot of space and decrease the chances of errors.

Scientific Notation Rules

We must follow the five rules when writing numbers in scientific notation:

  • The base should always be 10
  • The exponent (n) must be a non-zero integer, positive or negative
  • The absolute value of the coefficient (a) is greater than or equal to 1, but it should be less than 10 (1 ≤ a < 10)
  • The coefficient (a) can be positive or negative numbers, including whole numbers and decimal numbers
  • The mantissa contains the remaining significant digits of the number

How to Do Scientific Notation with Examples

As we know, in scientific notation, there are two parts:

  • Part 1: Consisting of just the digits with the decimal point placed after the first digit
  • Part 2: This part follows the first part by × 10 to a power that puts the decimal point where it should be

While writing numbers in scientific notation, we need to figure out how many places we should move the decimal point. The exponent of 10 determines the number of places the decimal point gets shifted to represent the number in long form.

There are two possibilities:

scientific notation problem solving questions

Case 1: With Positive Exponent

When the non-zero digit is followed by a decimal point

For example, if we want to represent 4237.8 in scientific notation, it will be:

  • The first part will be 4.2378 (only the digit and the decimal point placed after the first digit)
  • The second part following the first part will be × 10 3 (multiplied by 10 having a power of 3)

Case 2: With Negative Exponent

When the decimal point comes first, and the non-zero digit comes next

For example, if we want to represent 0.000082 in scientific notation, it will be:

  • The first part will be 8.2 (only the coefficient in decimal form and the decimal point placed after the first digit)
  • The second part following the first part will be × 10 -5 (multiplied by 10 having a power of -5)

Here is a table showing some more examples of numbers written in scientific notation:

Let us solve some more word problems involving writing numbers in scientific notation.

Write the number 0.0065 in scientific notation.

0.0065 is written in scientific notation as: 6.5 × 10 -3

Convert 4.5 in scientific notation.

4.5 is written in scientific notation as: 4.5 × 10 0

Write 53010000 in scientific notation.

53010000 is written in scientific notation as: 5.301 ×10 7

Light travels with a speed of 1.86 x 10 5 miles/second. It takes sunlight 4.8 x 10 3 seconds to reach Saturn. Find the approximate distance between Sun and Saturn. Express your answer in scientific notation.

As we know, Distance (d) = Speed (s) × Time (t), here s = 1.86 x 10 5 miles/second, t = 4.8 x 10 3 seconds = 8.928 x 10 8 miles

Other Ways of Writing in Scientific Notation

We sometimes use the ^ symbol instead of power while writing numbers in scientific notation. In such cases, the above number 4237.8, written in scientific notation as 4.2378 × 10 3 , can also be written as 4.2378 × 10^3 Similarly, calculators use the notation 4.2378E; here, E signifies 10 × 10 × 10

  • Converting Scientific Notation to Standard Form
  • Multiplying Numbers in Scientific Notation
  • Dividing Numbers in Scientific Notation
  • Adding and Subtracting Numbers in Scientific Notation

Leave a Reply Cancel reply

Your email address will not be published. Required fields are marked *

Save my name, email, and website in this browser for the next time I comment.

About us Contact us

  • Trigonometry

Join Our Newsletter

© 2024 Mathmonks.com . All rights reserved. Reproduction in whole or in part without permission is prohibited.

Module 7: Exponents

Scientific notation, learning objectives.

  • Define decimal and scientific notation

Convert between scientific and decimal notation

  • Multiply and divide numbers expressed in scientific notation

Problem Solving With Scientific Notation

In the same way that exponents help us to be able to write repeated multiplication with little effort, they are also used to express large and small numbers without a lot of zeros and confusion. Scientists and engineers make use of exponents regularly to keep trace of the place value of numbers that they are working with to make calculations.

Before we can convert between scientific and decimal notation, we need to know the difference between the two.  S cientific notation is used by scientists, mathematicians, and engineers when they are working with very large or very small numbers. Using exponential notation, large and small numbers can be written in a way that is easier to read.

When a number is written in scientific notation, the exponent tells you if the term is a large or a small number. A positive exponent indicates a large number and a negative exponent indicates a small number that is between 0 and 1. It is difficult to understand just how big a billion or a trillion is. Here is a way to help you think about it.

1 billion can be written as 1,000,000,000 or represented as [latex]10^9[/latex]. How would 2 billion be represented? Since 2 billion is 2 times 1 billion, then 2 billion can be written as [latex]2\times10^9[/latex].

A light year is the number of miles light travels in one year, about 5,880,000,000,000.  That’s a lot of zeros, and it is easy to lose count when trying to figure out the place value of the number. Using scientific notation, the distance is [latex]5.88\times10^{12}[/latex] miles. The exponent of 12 tells us how many places to count to the left of the decimal. Another example of how scientific notation can make numbers easier to read is the diameter of a hydrogen atom, which is about 0.00000005 mm, and in scientific notation is [latex]5\times10^{-8}[/latex] mm. In this case the [latex]-8[/latex] tells us how many places to count to the right of the decimal.

Outlined in the box below are some important conventions of scientific notation format.

A positive number is written in scientific notation if it is written as [latex]a\times10^{n}[/latex] where the coefficient a  is [latex]1\leq{a}<10[/latex], and n is an integer.

Look at the numbers below. Which of the numbers is written in scientific notation?

Now let’s compare some numbers expressed in both scientific notation and standard decimal notation in order to understand how to convert from one form to the other. Take a look at the tables below. Pay close attention to the exponent in the scientific notation and the position of the decimal point in the decimal notation.

0.05[latex]5\times10^{-2}[/latex]  0.0008[latex]8\times10^{-4}[/latex]  0.00000043[latex]4.3\times10^{-7}[/latex]  0.000000000625[latex]6.25\times10^{-10}[/latex]

Convert from decimal notation to scientific notation

To write a large number in scientific notation, move the decimal point to the left to obtain a number between 1 and 10. Since moving the decimal point changes the value, you have to multiply the decimal by a power of 10 so that the expression has the same value.

Let’s look at an example.

[latex]\begin{array}{r}180,000.=18,000.0\times10^{1}\\1,800.00\times10^{2}\\180.000\times10^{3}\\18.0000\times10^{4}\\1.80000\times10^{5}\\180,000=1.8\times10^{5}\end{array}[/latex]

Notice that the decimal point was moved 5 places to the left, and the exponent is 5.

Write the following numbers in scientific notation.

  • [latex]920,000,000[/latex]
  • [latex]10,200,000[/latex]
  • [latex]100,000,000,000[/latex]
  • [latex]\underset{\longleftarrow}{920,000,000}[/latex]  We will move the decimal point to the left, it helps to place it at the end of the number and then count how many times you move it to get one number before it that is between 1 and 10. [latex]\underset{\longleftarrow}{920,000,000}=920,000,000.0[/latex], move the decimal point 8 times to the left and you will have [latex]9.20,000,000[/latex], now we can replace the zeros with an exponent of 8, [latex]9.2\times10^{8}[/latex]
  • [latex]\underset{\longleftarrow}{10,200,000}=10,200,000.0=1.02\times10^{7}[/latex], note here how we included the 0 and the 2 after the decimal point.  In some disciplines, you may learn about when to include both of these.  Follow instructions from your teacher on rounding rules.
  • [latex]\underset{\longleftarrow}{100,000,000,000}=100,000,000,000.0=1.0\times10^{11}[/latex]

To write a small number (between 0 and 1) in scientific notation, you move the decimal to the right and the exponent will have to be negative,  as in the following example.

[latex]\begin{array}{r}\underset{\longrightarrow}{0.00004}=00.0004\times10^{-1}\\000.004\times10^{-2}\\0000.04\times10^{-3}\\00000.4\times10^{-4}\\000004.\times10^{-5}\\0.00004=4\times10^{-5}\end{array}[/latex]

You may notice that the decimal point was moved five places to the right until you got  to the number 4, which is between 1 and 10. The exponent is [latex]−5[/latex].

  • [latex]0.0000000000035[/latex]
  • [latex]0.0000000102[/latex]
  • [latex]0.00000000000000793[/latex]
  • [latex]\underset{\longrightarrow}{0.0000000000035}=3.5\times10^{-12}[/latex], we moved the decimal 12 times to get to a number between 1 and 10
  • [latex]\underset{\longrightarrow}{0.0000000102}=1.02\times10^{-8}[/latex]
  • [latex]\underset{\longrightarrow}{0.00000000000000793}=7.93\times10^{-15}[/latex]

In the following video you are provided with examples of how to convert both a large and a small number in decimal notation to scientific notation.

Convert from scientific notation to decimal notation

You can also write scientific notation as decimal notation. Recall the number of miles that light travels in a year is [latex]5.88\times10^{12}[/latex], and a hydrogen atom has a diameter of [latex]5\times10^{-8}[/latex] mm. To write each of these numbers in decimal notation, you move the decimal point the same number of places as the exponent. If the exponent is positive , move the decimal point to the right. If the exponent is negative , move the decimal point to the left.

[latex]\begin{array}{l}5.88\times10^{12}=\underset{\longrightarrow}{5.880000000000.}=5,880,000,000,000\\5\times10^{-8}=\underset{\longleftarrow}{0.00000005.}=0.00000005\end{array}[/latex]

For each power of 10, you move the decimal point one place. Be careful here and don’t get carried away with the zeros—the number of zeros after the decimal point will always be 1 less than the exponent because it takes one power of 10 to shift that first number to the left of the decimal.

Write the following in decimal notation.

  • [latex]4.8\times10{-4}[/latex]
  • [latex]3.08\times10^{6}[/latex]
  • [latex]4.8\times10^{-4}[/latex], the exponent is negative, so we need to move the decimal to the left.  [latex]\underset{\longleftarrow}{4.8\times10^{-4}}=\underset{\longleftarrow}{.00048}[/latex]
  • [latex]3.08\times10^{6}[/latex], the exponent is positive, so we need to move the decimal to the right.  [latex]\underset{\longrightarrow}{3.08\times10^{6}}=\underset{\longrightarrow}{3080000}[/latex]

Think About It

To help you get a sense of the relationship between the sign of the exponent and the relative size of a number written in scientific notation, answer the following questions. You can use the textbox to write your ideas before you reveal the solution.

1. You are writing a number whose absolute value is greater than 1 in scientific notation.  Will your exponent be positive or negative?

2.You are writing a number whose absolute value is between 0 and 1 in scientific notation.  Will your exponent be positive or negative?

3. What power do you need to put on 10 to get a result of 1?

2. You are writing a number whose absolute value    is between 0 and 1 in scientific notation. Will your exponent be positive or negative? We can reason that since numbers greater than 1 will have a positive exponent, numbers between 0 and 1 will have a negative exponent. Why are we specifying numbers between 0 and 1? The numbers between 0 and 1 represent amounts that are fractional. Recall that we defined numbers with a negative exponent as [latex]{a}^{-n}=\frac{1}{{a}^{n}}[/latex], so if we have [latex]10^{-2}[/latex] we have [latex]\frac{1}{10\times10}=\frac{1}{100}[/latex] which is a number between 0 and 1.

3.  What power do you need to put on 10 to get a result of 1? Recall that any number or variable with an exponent of 0 is equal to 1, as in this example:

[latex]\begin{array}{c}\frac{t^{8}}{t^{8}}=\frac{\cancel{t^{8}}}{\cancel{t^{8}}}=1\\\frac{{t}^{8}}{{t}^{8}}={t}^{8 - 8}={t}^{0}\\\text{ therefore }\\{t}^{0}=1\end{array}\\[/latex]

We now have described the notation necessary to write all possible numbers on the number line in scientific notation.

In the next video you will see how to convert a number written in scientific notation into decimal notation.

Large and small numbers can be written in scientific notation to make them easier to understand. In the next section, you will see that performing mathematical operations such as multiplication and division on large and small numbers is made easier by scientific notation and the rules of exponents.

Multiply and Divide Numbers in Scientific Notation

Numbers that are written in scientific notation can be multiplied and divided rather simply by taking advantage of the properties of numbers and the rules of exponents that you may recall. To multiply numbers in scientific notation, first multiply the numbers that aren’t powers of 10 (the a in [latex]a\times10^{n}[/latex]). Then multiply the powers of ten by adding the exponents.

This will produce a new number times a different power of 10. All you have to do is check to make sure this new value is in scientific notation. If it isn’t, you convert it.

Let’s look at some examples.

[latex]\left(3\times10^{8}\right)\left(6.8\times10^{-13}\right)[/latex]

[latex]\left(3\times6.8\right)\left(10^{8}\times10^{-13}\right)[/latex]

Multiply the coefficients.

[latex]\left(20.4\right)\left(10^{8}\times10^{-13}\right)[/latex]

Multiply the powers of 10 using the Product Rule. Add the exponents.

[latex]20.4\times10^{-5}[/latex]

Convert 20.4 into scientific notation by moving the decimal point one place to the left and multiplying by [latex]10^{1}[/latex].

[latex]\left(2.04\times10^{1}\right)\times10^{-5}[/latex]

Group the powers of 10 using the associative property of multiplication.

[latex]2.04\times\left(10^{1}\times10^{-5}\right)[/latex]

Multiply using the Product Rule—add the exponents.

[latex]2.04\times10^{1+\left(-5\right)}[/latex]

[latex]\left(3\times10^{8}\right)\left(6.8\times10^{-13}\right)=2.04\times10^{-4}[/latex]

[latex]\left(8.2\times10^{6}\right)\left(1.5\times10^{-3}\right)\left(1.9\times10^{-7}\right)[/latex]

[latex]\left(8.2\times1.5\times1.9\right)\left(10^{6}\times10^{-3}\times10^{-7}\right)[/latex]

Multiply the numbers.

[latex]\left(23.37\right)\left(10^{6}\times10^{-3}\times10^{-7}\right)[/latex]

Multiply the powers of 10 using the Product Rule—add the exponents.

[latex]23.37\times10^{-4}[/latex]

Convert 23.37 into scientific notation by moving the decimal point one place to the left and multiplying by [latex]10^{1}[/latex].

[latex]\left(2.337\times10^{1}\right)\times10^{-4}[/latex]

[latex]2.337\times\left(10^{1}\times10^{-4}\right)[/latex]

Multiply using the Product Rule and add the exponents.

[latex]2.337\times10^{1+\left(-4\right)}[/latex]

[latex]\left(8.2\times10^{6}\right)\left(1.5\times10^{-3}\right)\left(1.9\times10^{-7}\right)=2.337\times10^{-3}[/latex]

In the following video you will see an example of how to multiply tow numbers that are written in scientific notation.

In order to divide numbers in scientific notation, you once again apply the properties of numbers and the rules of exponents. You begin by dividing the numbers that aren’t powers of 10 (the a in [latex]a\times10^{n}[/latex]. Then you divide the powers of ten by subtracting the exponents.

This will produce a new number times a different power of 10. If it isn’t already in scientific notation, you convert it, and then you’re done.

[latex] \displaystyle \frac{2.829\times 1{{0}^{-9}}}{3.45\times 1{{0}^{-3}}}[/latex]

[latex] \displaystyle \left( \frac{2.829}{3.45} \right)\left( \frac{{{10}^{-9}}}{{{10}^{-3}}} \right)[/latex]

Divide the coefficients.

[latex] \displaystyle \left(0.82\right)\left( \frac{{{10}^{-9}}}{{{10}^{-3}}} \right)[/latex]

Divide the powers of 10 using the Quotient Rule. Subtract the exponents.

[latex]\begin{array}{l}0.82\times10^{-9-\left(-3\right)}\\0.82\times10^{-6}\end{array}[/latex]

Convert 0.82 into scientific notation by moving the decimal point one place to the right and multiplying by [latex]10^{-1}[/latex].

[latex]\left(8.2\times10^{-1}\right)\times10^{-6}[/latex]

Group the powers of 10 together using the associative property.

[latex]8.2\times\left(10^{-1}\times10^{-6}\right)[/latex]

[latex]8.2\times10^{-1+\left(-6\right)}[/latex]

[latex] \displaystyle \frac{2.829\times {{10}^{-9}}}{3.45\times {{10}^{-3}}}=8.2\times {{10}^{-7}}[/latex]

Example (Advanced)

[latex] \displaystyle \frac{\left(1.37\times10^{4}\right)\left(9.85\times10^{6}\right)}{5.0\times10^{12}}[/latex]

[latex] \displaystyle \frac{\left( 1.37\times 9.85 \right)\left( {{10}^{6}}\times {{10}^{4}} \right)}{5.0\times {{10}^{12}}}[/latex]

[latex] \displaystyle \frac{13.4945\times {{10}^{10}}}{5.0\times {{10}^{12}}}[/latex]

Regroup using the associative property.

[latex] \displaystyle \left( \frac{13.4945}{5.0} \right)\left( \frac{{{10}^{10}}}{{{10}^{12}}} \right)[/latex]

Divide the numbers.

[latex] \displaystyle \left(2.6989\right)\left(\frac{10^{10}}{10^{12}}\right)[/latex]

Divide the powers of 10 using the Quotient Rule—subtract the exponents.

[latex] \displaystyle \begin{array}{c}\left(2.6989 \right)\left( {{10}^{10-12}} \right)\\2.6989\times {{10}^{-2}}\end{array}[/latex]

[latex] \displaystyle \frac{\left( 1.37\times {{10}^{4}} \right)\left( 9.85\times {{10}^{6}} \right)}{5.0\times {{10}^{12}}}=2.6989\times {{10}^{-2}}[/latex]

Notice that when you divide exponential terms, you subtract the exponent in the denominator from the exponent in the numerator. You will see another example of dividing numbers written in scientific notation in the following video.

Molecule of water with one oxygen bonded to two hydrogen.

Water Molecule

Learning rules for exponents seems pointless without context, so let’s explore some examples of using scientific notation that involve real problems. First, let’s look at an example of how scientific notation can be used to describe real measurements.

Match each length in the table with the appropriate number of meters described in scientific notation below. Write your ideas in the textboxes provided before you look at the solution.

Red Blood Cells.

Red Blood Cells

One of the most important parts of solving a “real” problem is translating the words into appropriate mathematical terms, and recognizing when a well known formula may help. Here’s an example that requires you to find the density of a cell, given its mass and volume. Cells aren’t visible to the naked eye, so their measurements, as described with scientific notation, involve negative exponents.

Human cells come in a wide variety of shapes and sizes. The mass of an average human cell is about [latex]2\times10^{-11}[/latex] grams [1] Red blood cells are one of the smallest types of cells [2] , clocking in at a volume of approximately [latex]10^{-6}\text{ meters }^3[/latex]. [3] Biologists have recently discovered how to use the density of some types of cells to indicate the presence of disorders such as sickle cell anemia or leukemia. [4]  Density is calculated as the ratio of [latex]\frac{\text{ mass }}{\text{ volume }}\\[/latex]. Calculate the density of an average human cell.

Read and Understand:  We are given an average cellular mass and volume as well as the formula for density. We are looking for the density of an average human cell.

Define and Translate:   [latex]m=\text{mass}=2\times10^{-11}[/latex], [latex]v=\text{volume}=10^{-6}\text{ meters}^3\\[/latex], [latex]\text{density}=\frac{\text{ mass }}{\text{ volume }}\\[/latex]

Write and Solve:  Use the quotient rule to simplify the ratio.

[latex]\begin{array}{c}\text{ density }=\frac{2\times10^{-11}\text{ grams }}{10^{-6}\text{ meters }^3}\\\text{ }\\\,\,\,\,\,\,\,\,\,\,\,\,\,\,\,\,\,\,\,\,\,\,\,\,\,\,\,\,\,\,\,\,\,\,\,\,\,=2\times10^{-11-\left(-6\right)}\frac{\text{ grams }}{\text{ meters }^3}\\\text{ }\\\,\,\,\,\,\,\,\,\,\,\,\,\,\,\,\,\,\,\,\,\,\,\,=2\times10^{-5}\frac{\text{ grams }}{\text{ meters }^3}\\\end{array}[/latex]

If scientists know the density of healthy cells, they can compare the density of a sick person’s cells to that to rule out or test for disorders or diseases that may affect cellular density.

The average density of a human cell is [latex]2\times10^{-5}\frac{\text{ grams }}{\text{ meters }^3}[/latex]

The following video provides an example of how to find the number of operations a computer can perform in a very short amount of time.

Earth in the foreground, sun in the background, light beams traveling from the sun to the earth.

Light traveling from the sun to the earth.

In the next example, you will use another well known formula, [latex]d=r\cdot{t}[/latex], to find how long it takes light to travel from the sun to the earth. Unlike the previous example, the distance between the earth and the sun is massive, so the numbers you will work with have positive exponents.

The speed of light is [latex]3\times10^{8}\frac{\text{ meters }}{\text{ second }}\\[/latex]. If the sun is [latex]1.5\times10^{11}[/latex] meters from earth, how many seconds does it take for sunlight to reach the earth?  Write your answer in scientific notation.

Read and Understand:  We are looking for how long—an amount of time. We are given a rate which has units of meters per second and a distance in meters. This is a [latex]d=r\cdot{t}[/latex] problem.

Define and Translate: 

[latex]\begin{array}{l}d=1.5\times10^{11}\\r=3\times10^{8}\frac{\text{ meters }}{\text{ second }}\\t=\text{ ? }\end{array}\\[/latex]

Write and Solve:  Substitute the values we are given into the [latex]d=r\cdot{t}[/latex] equation. We will work without units to make it easier. Often, scientists will work with units to make sure they have made correct calculations.

[latex]\begin{array}{c}d=r\cdot{t}\\1.5\times10^{11}=3\times10^{8}\cdot{t}\end{array}[/latex]

Divide both sides of the equation by [latex]3\times10^{8}[/latex] to isolate  t.

[latex]\begin{array}{c}1.5\times10^{11}=3\times10^{8}\cdot{t}\\\text{ }\\\frac{1.5\times10^{11}}{3\times10^{8}}=\frac{3\times10^{8}}{3\times10^{8}}\cdot{t}\end{array}[/latex]

On the left side, you will need to use the quotient rule of exponents to simplify, and on the right, you are left with  t. 

[latex]\begin{array}{c}\frac{1.5\times10^{11}}{3\times10^{8}}=\frac{3\times10^{8}}{3\times10^{8}}\cdot{t}\\\text{ }\\\left(\frac{1.5}{3}\right)\times\left(\frac{10^{11}}{10^{8}}\right)=t\\\text{ }\\\left(0.5\right)\times\left(10^{11-8}\right)=t\\0.5\times10^3=t\end{array}[/latex]

This answer is not in scientific notation, so we will move the decimal to the right, which means we need to subtract one factor of 10.

[latex]0.5\times10^3=5.0\times10^2=t[/latex]

The time it takes light to travel from the sun to the earth is [latex]5.0\times10^2=t[/latex] seconds, or in standard notation, 500 seconds.  That’s not bad considering how far it has to travel!

In the following video you will see that the total number of miles run by the participants in the Boston marathon is larger than the circumference of the earth! https://youtu.be/san2avgwu6k

Scientific notation was developed to assist mathematicians, scientists, and others when expressing and working with very large and very small numbers. Scientific notation follows a very specific format in which a number is expressed as the product of a number greater than or equal to one and less than ten, and a power of 10. The format is written [latex]a\times10^{n}[/latex], where [latex]1\leq{a}<10[/latex] and n is an integer. To multiply or divide numbers in scientific notation, you can use the commutative and associative properties to group the exponential terms together and apply the rules of exponents.

  • Orders of magnitude (mass). (n.d.). Retrieved May 26, 2016, from https://en.wikipedia.org/wiki/Orders_of_magnitude_(mass) ↵
  • How Big is a Human Cell? ↵
  • How big is a human cell? - Weizmann Institute of Science. (n.d.). Retrieved May 26, 2016, from http://www.weizmann.ac.il/plants/Milo/images/humanCellSize120116Clean.pdf ↵
  • Grover, W. H., Bryan, A. K., Diez-Silva, M., Suresh, S., Higgins, J. M., & Manalis, S. R. (2011). Measuring single-cell density. Proceedings of the National Academy of Sciences, 108(27), 10992-10996. doi:10.1073/pnas.1104651108 ↵
  • Revision and Adaptation. Provided by : Lumen Learning. License : CC BY: Attribution
  • Unit 11: Exponents and Polynomials, from Developmental Math: An Open Program. Provided by : Monterey Institute of Technology and Education. Located at : http://nrocnetwork.org/dm-opentext . License : CC BY: Attribution
  • Examples: Write a Number in Scientific Notation. Authored by : James Sousa (Mathispower4u.com) for Lumen Learning. Located at : https://youtu.be/fsNu3AdIgdk . License : CC BY: Attribution
  • Examples: Writing a Number in Decimal Notation When Given in Scientific Notation. Authored by : James Sousa (Mathispower4u.com) for Lumen Learning. Located at : https://youtu.be/8BX0oKUMIjw . License : CC BY: Attribution
  • Examples: Dividing Numbers Written in Scientific Notation. Authored by : James Sousa (Mathispower4u.com) . Located at : https://youtu.be/8BX0oKUMIjw . License : CC BY: Attribution
  • Examples: Multiplying Numbers Written in Scientific Notation. Authored by : James Sousa (Mathispower4u.com) for Lumen Learning. Located at : https://youtu.be/5ZAY4OCkp7U . License : CC BY: Attribution

Library homepage

  • school Campus Bookshelves
  • menu_book Bookshelves
  • perm_media Learning Objects
  • login Login
  • how_to_reg Request Instructor Account
  • hub Instructor Commons
  • Download Page (PDF)
  • Download Full Book (PDF)
  • Periodic Table
  • Physics Constants
  • Scientific Calculator
  • Reference & Cite
  • Tools expand_more
  • Readability

selected template will load here

This action is not available.

Chemistry LibreTexts

1.7: Expressing Numbers - Scientific Notation

  • Last updated
  • Save as PDF
  • Page ID 406491

Learning Objectives

  • Express a large number or a small number in scientific notation.
  • Convert a number in scientific notation to standard conventional form.

The instructions for making a pot of coffee specified 3 scoops (rather than 12,000 grounds) because any measurement is expressed more efficiently with units that are appropriate in size. In science, however, we often must deal with quantities that are extremely small or incredibly large. For example, you may have 5,000,000,000,000 red blood cells in a liter of blood, and the diameter of an iron atom is 0.000000014 inches. Numbers with many zeros can be cumbersome to work with, so scientists use scientific notation.

Scientific notation is a system for expressing very large or very small numbers in a compact manner. It uses the idea that such numbers can be rewritten as a simple number multiplied by 10 raised to a certain exponent, or power.

Let us look first at very large numbers. Suppose a spacecraft is 1,500,000 miles from Mars. The number 1,500,000 can be thought of as follows:

clipboard_ef189d70a936cb5c9d025a8ac1fea3be6.png

That is, 1,500,000 is the same as 1.5 times 1 million, and 1 million is 10 × 10 × 10 × 10 × 10 × 10, or 10 6 (which is read as “ten to the sixth power”). Therefore, 1,500,000 can be rewritten as 1.5 times 10 6 , or 1.5 × 10 6 . The distance of the spacecraft from Mars can therefore be expressed as 1.5 × 10 6 miles.

Recall that:

  • 10 3 = 1,000
  • 10 4 = 10,000
  • and so forth

The standard convention for expressing numbers in scientific notation is to write a single nonzero first digit, a decimal point, and the rest of the digits, excluding any trailing zeros (see rules for significant figures in the next section for more details on what to exclude). This number is followed by a multiplication sign and then by 10 raised to the power necessary to reproduce the original number. For example, although 1,500,000 can also be written as 15. × 10 5 (which would be 15. × 100,000), the convention is to have only one digit before the decimal point. How do we know to what power 10 is raised? The power is the number of places you have to move the decimal point to the left to place it after the first digit , so that the number being multiplied is between 1 and 10 :

1,500,000 = 1.5 times 10 to the sixth.

Example \(\PageIndex{1}\): Scientific Notation

Express each number in scientific notation.

  • 67,000,000,000

Moving the decimal point 10 places to the left gives 6.7 × 10 10 .

The decimal point is assumed to be at the end of the number, so moving it three places to the left gives 1.689 × 10 3 .

In this case, we need to move the decimal point only one place to the left, which yields 1.26 × 10 1 .

Exercise \(\PageIndex{1}\)

  • 102,000,000

Moving the decimal point 3 places to the left gives 1.492 × 10 3 .

The decimal point is assumed to be at the end of the number, so moving it 8 places to the left gives 1.02 × 10 8 .

Moving the decimal point 5 places to the left yields 1.01325 × 10 5 .

To change a number in scientific notation to standard form , we reverse the process, moving the decimal point to the right. Add zeros to the end of the number being converted, if necessary, to produce a number of the proper magnitude. Lastly, we drop the number 10 and its power.

1.5 times 10 to the sixth = 1.500000 with an arrow bringing the decimal point to the end = 1,500,000

Example \(\PageIndex{2}\)

Express each number in standard, or conventional notation.

  • 5.27 × 10 4
  • 1.0008 × 10 6

Moving the decimal four places to the right and adding zeros give 52,700.

Moving the decimal six places to the right and adding zeros give 1,000,800.

Exercise \(\PageIndex{2}\)

  • \(6.98 \times 10^8\)
  • \(1.005 \times 10^2\)

Moving the decimal point eight places to the right and adding zeros give 698,000,000.

Moving the decimal point two places to the right gives 100.5

We can also use scientific notation to express numbers whose magnitudes are less than 1. For example, the quantity 0.006 centimeters can be expressed as follows:

clipboard_e500cf8f05c67d765d316a398489d85f6.png

That is, 0.006 centimeters is the same as 6 divided by one thousand, which is the same as 6 divided by 10 x 10 x 10 or 6 times 10 –3 (which is read as "ten to the negative third power"). Therefore, 0.006 centimeters can be rewritten as 6 times 10 –3 , or 6 × 10 –3 centimeters.

  • 10 −1 = 1/10
  • 10 −2 = 1/100
  • 10 −3 = 1/1,000
  • 10 −4 = 1/10,000
  • 10 −5 = 1/100,000

We use a negative number as the power to indicate the number of places we have to move the decimal point to the right to make it follow the first nonzero digit so that the number is between 1 and 10. This is illustrated as follows:

0.006 with an arrow bringing the decimal to the end = 6 times 10 to the negative third power.

In writing scientific notations, the convention is to have only one digit before the decimal point.

1.5x10^6.v2.jpg

Example \(\PageIndex{3}\)

  • 0.000006567
  • −0.0004004
  • 0.000000000000123

Move the decimal point six places to the right to get 6.567 × 10 −6 .

Move the decimal point four places to the right to get −4.004 × 10 −4 . The negative sign on the number itself does not affect how we apply the rules of scientific notation.

Move the decimal point 13 places to the right to get 1.23 × 10 −13 .

Exercise \(\PageIndex{3}\)

  • −0.051204

Moving the decimal point four places to the right gives 3.55 × 10 −4 .

Moving the decimal point one place to the right gives 3.14159 × 10 −1 .

Moving the decimal point one place to the right gives −5.1204 × 10 −2 .

As with numbers with positive powers of 10, when changing from scientific notation to standard or conventional format, we reverse the process.

6 times 10 to the negative third = 006. with an arrow bringing the decimal to the front = 0.006

Changing a number in scientific notation to standard form:

  • If the scientific notation has a positive power, the standard number is greater than one. Example: 8 x 10 4 = 80,000
  • If the scientific notation has a negative power, then the standard number is less than one. Example: 8 x 10 -2 = 0.08

Example \(\PageIndex{4}\)

Change the number in scientific notation to standard form.

  • 6.22 × 10 −2
  • 9.9 × 10 −9

0.0000000099

Exercise \(\PageIndex{4}\)

  • 9.98 × 10 −5
  • 5.109 × 10 −8

0.00000005109

Although calculators can show 8 to 10 digits in their display windows, that is not always enough when working with very large or very small numbers. For this reason, many calculators are designed to handle scientific notation. The method for entering scientific notation differs for each calculator model, so take the time to learn how to do it properly on your calculator, asking your instructor for assistance if necessary . If you do not learn to enter scientific notation into your calculator properly, you will not get the correct final answer when performing a calculation.

Concept Review Exercises

  • Why it is easier to use scientific notation to express very large or very small numbers?
  • What is the relationship between how many places a decimal point moves and the power of 10 used in changing a conventional number into scientific notation?
  • Scientific notation is more convenient than listing a large number of zeros.
  • The number of places the decimal point moves equals the power of 10. The power of 10 is positive if the decimal point moves to the left and negative if the decimal point moves to the right.

Key Takeaway

  • Large or small numbers are expressed in scientific notation, which use powers of 10.

Scientific Notation Quiz

scientific notation problem solving questions

This online quiz is intended to give you extra practice in converting between scientific notation and decimal notation.

Select your preferences below and click 'Start' to give it a try!

scientific notation problem solving questions

If you're seeing this message, it means we're having trouble loading external resources on our website.

If you're behind a web filter, please make sure that the domains *.kastatic.org and *.kasandbox.org are unblocked.

To log in and use all the features of Khan Academy, please enable JavaScript in your browser.

Course: 8th grade   >   Unit 1

  • Scientific notation word problem: red blood cells
  • Scientific notation word problem: U.S. national debt

Scientific notation word problem: speed of light

  • Scientific notation word problems
  • Numbers and operations: FAQ

Want to join the conversation?

  • Upvote Button navigates to signup page
  • Downvote Button navigates to signup page
  • Flag Button navigates to signup page

Good Answer

Video transcript

SCIENTIFIC NOTATION WORD PROBLEMS WORKSHEET

Problem 1 :

The table below shows the population of the three largest countries in  North America in 2011. Find the total population of these countries.

scientific notation problem solving questions

Problem 2 :

When the Sun makes an orbit around the center of the Milky Way, it travels  2.025 x 10 14   kilometers. The orbit takes 225 million years. At what rate  does the Sun travel? Write your answer in scientific notation.

Problem 3 :

Light travels at a speed of 1.86 x 10 5  miles per second. It takes light from the Sun about 4.8 x 10 3  seconds to reach Saturn. Find the approximate distance from the Sun to Saturn. Write your answer in scientific notation.

Problem 4 :

Light travels at the speed of 1.17 x 10 7  miles per minute. Pluto’s average distance from the Sun is 3,670,000,000 miles. On average, how long does it take sunlight to reach Pluto? Write your answer in scientific notation.

scientific notation problem solving questions

Detailed Answer Key

Step 1 : 

First, write each population with the same power of 10.

United States : 3.1 x 10 8

Canada : 0.338 x  10 8

Mexico : 1.1 x  10 8

Step 2 : 

Add the multipliers for each population.

3.1 + 0.338 + 1.1  =  4.538

Step 3 : 

Write the final answer in scientific notation :

4.538 x  10 8

First, write each number in standard notation.

United States :  310,000,000

Canada :  33,800,000

Mexico :  110,000,000

Find the sum of the numbers in standard notation.

310,000,000 + 33,800,000 + 110,000,000  =  453,800,000

453,800,000  =   4.538 x  10 8

Key points : 

The answer is the number of kilometers per year that the Sun travels  around the Milky Way.

Set up a division problem using

Rate  =  Distance / Time 

to represent the situation.

Substitute the values from the problem into the Rate formula. 

scientific notation problem solving questions

Write the expression for rate with years in scientific notation.

That is, 225 million  =  2.25 x  10 8 .

Then, we have

scientific notation problem solving questions

Find the quotient by dividing the decimals and using the laws of exponents.

Divide the multipliers.

2.025 ÷  2.25  =  0.9 

Divide the powers of 10.

10 14  ÷  10 8   =   10 14-8  

10 14  ÷  10 8   =   10 6  

Combine the answers to write the rate in scientific notation.

0.9 x  10 6    =  9.0 x 10 5

Justify and Evaluate : 

Use estimation to check the reasonableness of your answer.

scientific notation problem solving questions

9.0 x 10 5  is close 10 6 ,  so the answer is reasonable.

As we have solved question 2, we can solve get answer for this question using the formula for distance given below. 

Distance  =  Speed x Time

Then, we will get the answer : 

8.928 x 10 8  miles

As we have solved question 2, we can solve get answer for this question using the formula for time given below. 

Time   =  Distance / Speed  

3.14 x 10 2 minutes

scientific notation problem solving questions

Apart from the stuff given above, if you need any other stuff in math, please use our google custom search here.

Kindly mail your feedback to   [email protected]

We always appreciate your feedback.

© All rights reserved. onlinemath4all.com

  • Sat Math Practice
  • SAT Math Worksheets
  • PEMDAS Rule
  • BODMAS rule
  • GEMDAS Order of Operations
  • Math Calculators
  • Transformations of Functions
  • Order of rotational symmetry
  • Lines of symmetry
  • Compound Angles
  • Quantitative Aptitude Tricks
  • Trigonometric ratio table
  • Word Problems
  • Times Table Shortcuts
  • 10th CBSE solution
  • PSAT Math Preparation
  • Privacy Policy
  • Laws of Exponents

Recent Articles

RSS

Complex Plane

Apr 14, 24 07:56 AM

complexplane1.png

Real Analysis

Apr 14, 24 02:50 AM

How to Sketch the Graph of Sinusoidal Function

Apr 13, 24 12:02 PM

sinusoidalfunction3.png

  • Math Article
  • Scientific Notation

Scientific notation

Scientific notation is a form of presenting very large numbers or very small numbers in a simpler form. As we know, the whole numbers can be extended till infinity, but we cannot write such huge numbers on a piece of paper. Also, the numbers which are present at the millions place after the decimal needed to be represented in a simpler form. Thus, it is difficult to represent a few numbers in their expanded form. Hence, we use scientific notations. Also learn, Numbers In General Form .

Scientific notation

For example, 100000000 can be written as 10 8 , which is the scientific notation. Here the exponent is positive. Similarly, 0.0000001 is a very small number which can be represented as 10 -8 , where the exponent is negative.

Scientific Notation Definition

As discussed in the introduction, the scientific notation helps us to represent the numbers which are very huge or very tiny in a form of multiplication of single-digit numbers and 10 raised to the power of the respective exponent. The exponent is positive if the number is very large and it is negative if the number is very small. Learn power and exponents for better understanding.

The general representation of scientific notation is:

Also, read:

  • Scientific notation formula calculator
  • Scientific Notation Calculator

Scientific Notation Rules

To determine the power or exponent of 10,  we must follow the rule listed below:

  • The base should be always 10
  • The exponent must be a non-zero integer, that means it can be either positive or negative
  • The absolute value of the coefficient is greater than or equal to 1 but it should be less than 10
  •  Coefficients can be positive or negative numbers including whole and decimal numbers
  • The mantissa carries the rest of the significant digits of the number

Let us understand how many places we need to move the decimal point after the single-digit number with the help of the below representation.

  • If the given number is multiples of 10 then the decimal point has to move to the left, and the power of 10 will be positive. Example: 6000 = 6 × 10 3 is in scientific notation.
  • If the given number is smaller than 1, then the decimal point has to move to the right, so the power of 10 will be negative. Example: 0.006 = 6 × 0.001 = 6 × 10 -3  is in scientific notation.

Scientific Notation Examples

The examples of scientific notation are: 490000000 = 4.9×10 8 1230000000 = 1.23×10 9 50500000 = 5.05 x 10 7 0.000000097 = 9.7 x 10 -8 0.0000212 = 2.12 x 10 -5

Positive and Negative Exponent

When the scientific notation of any large numbers is expressed, then we use positive exponents for base 10. For example: 20000 = 2 x 10 4 , where 4 is the positive exponent.

When the scientific notation of any small numbers is expressed, then we use negative exponents for base 10. For example: 0.0002 = 2 x 10 -4 , where -4 is the negative exponent.

From the above, we can say that the number greater than 1 can be written as the expression with positive exponent, whereas the numbers less than 1 with negative exponent.

Problems and Solutions

Question 1: Convert 0.00000046 into scientific notation.

Solution: Move the decimal point to the right of 0.00000046 up to 7 places.

The decimal point was moved 7 places to the right to form the number 4.6

Since the numbers are less than 10 and the decimal is moved to the right. Hence, we use a negative exponent here.

⇒ 0.00000046 = 4.6 × 10 -7

This is the scientific notation.

Question 2: Convert 301000000 in scientific notation.

Solution: Move the decimal to the left 8 places so it is positioned to the right of the leftmost non zero digits 3.01000000. Remove all the zeroes and multiply the number by 10.

Now the number has become = 3.01.

Since the number is greater than 10 and the decimal is moved to left, therefore, we use here a positive exponent.

Hence, 3.01 × 10 8 is the scientific notation of the number.

Question 3:Convert 1.36 × 10 7 from scientific notation to standard notation.

Solution: Given, 1.36 × 10 7 in scientific notation.

Exponent = 7

Since the exponent is positive we need to move the decimal place 7 places to the right.

1.36 × 10 7 = 1.36 × 10000000 = 1,36,00,000.

Practice Questions

Problem 1: Convert the following numbers into scientific notation.

Problem 2: Convert the following into standard form.

  • 2.89 × 10 -6
  • 9.8 × 10 -2

Frequently Asked Questions on Scientific Notation – FAQs

How do you write 0.00001 in scientific notation, what are the 5 rules of scientific notation, what are the 3 parts of a scientific notation, how do you write 75 in scientific notation, how do you put scientific notation into standard form.

Quiz Image

Put your understanding of this concept to test by answering a few MCQs. Click ‘Start Quiz’ to begin!

Select the correct answer and click on the “Finish” button Check your score and answers at the end of the quiz

Visit BYJU’S for all Maths related queries and study materials

Your result is as below

Request OTP on Voice Call

scientific notation problem solving questions

  • Share Share

Register with BYJU'S & Download Free PDFs

Register with byju's & watch live videos.

close

Complete Test Preparation Inc.

Scientific Notation Math – Tutorial, Examples and Practice Questions

Scientific Notation

  • Posted by Brian Stocker MA
  • Date April 6, 2014
  • Comments 4 comments

Scientific Notation Math

Scientific notation is a way of writing numbers that are too big or too small to be conveniently written in decimal form.

Most High School and exams have scientific Notation questions

Audio Version of this Post

How to convert a number to scientific notation.

To convert a number to scientific notation, place a decimal after the first number that is not a zero, or, after the first number that between 1 and 9.

After placing the decimal, count the number of places the decimal had to move to get the exponent of 10. If the decimal moves to the left, then the exponent to multiply 10 will be in the positive. If the decimal moves from right to left, it will be a negative power of 10.

For example, to convert 29010, we need to place a decimal after 2, since 2 is the first non zero number. We would then have 2.90

If we were to convert 0.0167, we need to place the decimal after 1, since the first two numbers before 1 are zeros, and do not fall between 1 and 9. We would thus have 1.67

To complete the conversion of 29010 to scientific notation, we would get 2.91 x 10⁴

The 10 is raised to the power of 4, because there are 4 places counting from right to left. This scientific notation is positive because the decimal moved to the left.

0.0167 = 1.67 x 10 -2

In this example, the decimal place moved from left to right by 2 spaces thus the 10 is raised to the power of 2. It is negative, because the decimal moved to the right.

How to convert from scientific notation

You may also need to convert numbers that are already represented in scientific notation or in their power of ten, to regular numbers. First it is important to remember these two laws.

If the power is positive, shift decimal to the right If the power is negative, shift decimal point to the left

Scientific Notation Math – Practice Questions

1. Convert 7,892,000,000 to scientific notation.

A. 7.892 x 10 10 B. 7.892 x 10 -9 C. 7.892 x 10 9 D. 0.7892 x 10 11

2. Convert 0.045 to scientific notation.

A. 4.5 x 10 -2 B. 4.5 x 10 2 C. 4.05 x 10- 2 D. 4.5 x 10 -3

3. Convert 204 to scientific notation.

A. 2.04 x 10 -2 B. 0.204 x 10 2 C. 2.04 x 10 3 D. 2.04 x 10 2

4.  Convert 0.00002011 to scientific notation.

A. 2.011 x 10 -4 B. 2.011 x 10 5 C. 2.011 x 10 -6 D. 2.011 x 10 -5

5. Convert this scientific notation back to its original number: 2.63 x 10 -2

A. 0.00263 B. 0.0263 C. 0.263 D. 2.63

6. Convert this scientific notation back to its original number: 5.63 x 10 6

A. 5,630,000 B. 563,000 C. 5630 D. 0.000005.630

The decimal point moves 9 spaces right to be placed after 7, which is the first non-zero number. Thus 7.892 x 10 9

2. A The decimal point moves 2 spaces to the left to be placed before 4, which is the first non-zero number. Thus its 4.5 x 10 -2  The answer is in negative since the decimal moved left.

3. D The decimal point moves 2 spaces right to be placed after 2, which is the first non-zero number. Thus it is 2.04 x 10 2

4. D The decimal point moves 5 places left to be placed after 2, which is the first non-zero number. Thus its 2.011 x 10 -5  The answer is in the negative because the decimal moved left.

5. B The scientific notation is in the negative so we shift the decimal 2 places to the left. Thus its 0.0263.

6. A The scientific notation is in the positive so we shift the decimal 6 places to the right. Thus it is 5,630,000.

Common Mistakes on a Test with Scientific Notation

Not using the correct number of significant figures. This is the most common mistake!   Keep track of the correct number of significant figures.

Forgetting to use a multiplication symbol when converting from standard notation to scientific notation. Always use a multiplication symbol (e.g., 3 x 10^2 instead of 3 10^2) when working with scientific notation so that you keep everything clear.

Wrong placement of decimal points Also very common mistake on a test.  Place the decimal point so there is only one non-zero digit to the left of it.

Writing an incorrect exponent form The exponent to be a power of 10, written as 10^n.

Errors in rounding

Format errors Always use a consistent format for the exponent, and especially use the correct number of significant figures.

Don’t confuse scientific notation with standard notation Scientific notation is always to the power of 10.

Tag: Practice Questions , Scientific Notation

author avatar

Previous post

How to Solve Quadratic Equations - Tutorial, Examples and Practice Questions

Learn mean, median and mode - tutorial and practice questions, you may also like.

MathMC

Basic Math Video Tutorials

How to answer basic math multiple choice.

Inequality

How to Solve Linear Inequalities – Quick Review and Practice

' src=

Thanks for a convient website, loving it,keep up good work.

' src=

very very good questions. I solved all of them. thanks!

' src=

Please give harder questions

' src=

Not too difficult questions

Leave A Reply Cancel reply

Your email address will not be published. Required fields are marked *

IMAGES

  1. Operations With Scientific Notation Worksheet

    scientific notation problem solving questions

  2. Scientific Notation Practice 2

    scientific notation problem solving questions

  3. Scientific Notation Word Problems Independent Practice Worksheet

    scientific notation problem solving questions

  4. 40 scientific notation word problems worksheet with answers

    scientific notation problem solving questions

  5. Scientific Notation Worksheet Answer Key Scientific Notation Word

    scientific notation problem solving questions

  6. Scientific Notation Practice Worksheet

    scientific notation problem solving questions

VIDEO

  1. Year 9 Scientific Notation Example 5

  2. Math Olympiad

  3. Scientific Notation Made Super Easy

  4. Excel

  5. Scientific Notation: Simplify and write in scientific notation: 8,000,000 / 0.0002

  6. Scientific Notation Word Problems 127-4.12

COMMENTS

  1. Scientific notation (practice)

    Express this number in scientific notation. 0.3643. Learn for free about math, art, computer programming, economics, physics, chemistry, biology, medicine, finance, history, and more. Khan Academy is a nonprofit with the mission of providing a free, world-class education for anyone, anywhere.

  2. Scientific Notation

    These problems cover the fundamentals of writing scientific notation and using it to understand relative size of values and scientific prefixes. Problem 1: The distance to the moon is 238,900 miles. Write this value in scientific notation. Show me how. Problem 2: One mile is 1609.34 meters. What is the distance to the moon in meters using ...

  3. Scientific notation word problems (practice)

    Scientific notation word problems. Google Classroom. You might need: Calculator. Light travels 9.45 ⋅ 10 15 meters in a year. There are about 3.15 ⋅ 10 7 seconds in a year. How far does light travel per second?

  4. Problem Solving With Scientific Notation

    Scientific notation follows a very specific format in which a number is expressed as the product of a number greater than or equal to one and less than ten times a power of 10 10. The format is written a×10n a × 10 n, where 1 ≤a < 10 1 ≤ a < 10 and n is an integer. To multiply or divide numbers in scientific notation, you can use the ...

  5. 4.4: Scientific Notation

    Convert 0.82 into scientific notation by moving the decimal point one place to the right and multiplying by 10 − 1. (8.2 × 10 − 1) × 10 − 6. Group the powers of 10 together using the associative property. 8.2 × (10 − 1 × 10 − 6) Multiply the powers of 10 using the Product Rule—add the exponents.

  6. Scientific Notation Questions with Solutions

    Answer E. Solution. Scientific notation of a number is of form: a × 10n a × 10 n where 1 ≤ a < 10 1 ≤ a < 10 and n n is an integer. 510000 510000 is bigger than 10 10, therefore start with a decimal point from the right 510000. 510000. and move it till the number is between 1 1 and 10 10 not included.

  7. Solve problems using scientific notation

    Correct answer: 1.284 ×106. Explanation: You can solve this problem in several ways. One way is to convert each number out of scientific notation and write it out fully, then find the sum of the two values and convert the answer back into scientific notation. Another, potentially faster, way to solve this problem is to convert one answer into ...

  8. Scientific notation examples (video)

    Scientific notation examples. Scientific notation is a way of writing very large or very small numbers. A number is written in scientific notation when a number between 1 and 10 is multiplied by a power of 10. For example, 650,000,000 can be written in scientific notation as 6.5 10^8. Created by Sal Khan and CK-12 Foundation.

  9. Math Practice Problems

    Solution. Begin by finding out the significant part of the number. It should always be between 1 and 10. Thus, the significant part is 2.223. Must move the decimal 9 spots to the left to end up with the significant value. Thus, the exponential power should be 9. Scientific Notation = significant value × 10 exponent = 2.223*10^9. #.

  10. How to Solve Scientific Notation? (+FREE Worksheet!)

    Step by step guide to solve scientific notation problems. Scientific notation is used to write very big or very small numbers in decimal form. In scientific notation all numbers are written in the form of: \(m×10^n\) Scientific Notation

  11. Study Guide

    Scientific notation was developed to assist mathematicians, scientists, and others when expressing and working with very large and very small numbers. Scientific notation follows a very specific format in which a number is expressed as the product of a number greater than or equal to one and less than ten, and a power of 10 10.

  12. Problem Solving With Scientific Notation

    Scientific notation follows a very specific format in which a number is expressed as the product of a number greater than or equal to one and less than ten, and a power of 10. The format is written a×10n a × 10 n, where 1 ≤a <10 1 ≤ a < 10 and n is an integer. To multiply or divide numbers in scientific notation, you can use the ...

  13. Scientific Notation

    We must follow the five rules when writing numbers in scientific notation: The base should always be 10. The exponent (n) must be a non-zero integer, positive or negative. The absolute value of the coefficient (a) is greater than or equal to 1, but it should be less than 10 (1 ≤ a < 10) The coefficient (a) can be positive or negative numbers ...

  14. Adding & subtracting in scientific notation

    Learn for free about math, art, computer programming, economics, physics, chemistry, biology, medicine, finance, history, and more. Khan Academy is a nonprofit with the mission of providing a free, world-class education for anyone, anywhere. ... Express your answer in scientific notation. 8.2 ... Report a problem.

  15. Scientific Notation

    Problem Solving With Scientific Notation; ... a sense of the relationship between the sign of the exponent and the relative size of a number written in scientific notation, answer the following questions. You can use the textbox to write your ideas before you reveal the solution. 1. You are writing a number whose absolute value is greater than ...

  16. Scientific Notation: Practice Problems

    Problem solving - use acquired knowledge to solve scientific notation practice problems ... Knowledge application - use your knowledge to answer questions about the value of different equations

  17. Scientific Notation

    Scientific notation - Practice problems. After having reviewed the examples with answers, try to solve the following scientific notation problems. Simply choose an answer and select the "Check" button to check your chosen answer. If you are having trouble with these problems, you can look back at the above examples carefully.

  18. 1.7: Expressing Numbers

    Exercise 1.7.1 1.7. 1. Express each number in scientific notation. To change a number in scientific notation to standard form, we reverse the process, moving the decimal point to the right. Add zeros to the end of the number being converted, if necessary, to produce a number of the proper magnitude.

  19. Scientific Notation Quiz : ChemQuiz.net

    This online quiz is intended to give you extra practice in converting between scientific notation and decimal notation. Select your preferences below and click 'Start' to give it a try! Number of problems: 1. 5. 10. 25. 50. Type of questions:

  20. Scientific Notation: Practice Problems

    Scientific notation is a system for abbreviating very large or very small numbers. Instead of that whole mess of zeroes, you could just write: 1.6726 x 10^ -27 kg. This makes it much less likely ...

  21. Scientific notation word problem: speed of light

    Scientific notation word problem: speed of light. It is possible to simplify multiplication and division using scientific notation. This can be used to calculate the distance between the sun and the earth, which is 1.5 times 10 to the 11th power meters. This is an incredibly large distance and difficult to visualize.

  22. Scientific Notation Word Problems Worksheet

    Step 1 : Substitute the values from the problem into the Rate formula. Step 2 : Write the expression for rate with years in scientific notation. That is, 225 million = 2.25 x 108. Then, we have. Step 3 : Find the quotient by dividing the decimals and using the laws of exponents.

  23. Scientific notation

    Problems and Solutions. Question 1: Convert 0.00000046 into scientific notation. Solution: Move the decimal point to the right of 0.00000046 up to 7 places. The decimal point was moved 7 places to the right to form the number 4.6. Since the numbers are less than 10 and the decimal is moved to the right. Hence, we use a negative exponent here.

  24. Scientific Notation Math

    Forgetting to use a multiplication symbol when converting from standard notation to scientific notation. Always use a multiplication symbol (e.g., 3 x 10^2 instead of 3 10^2) when working with scientific notation so that you keep everything clear. Wrong placement of decimal points Also very common mistake on a test.